PA PROFESSION COMEBINED

Pataasin ang iyong marka sa homework at exams ngayon gamit ang Quizwiz!

What are four core requirements for CAQs?

In addition to general requirements }Four Core requirements: 1. Category 1 specialty CME (150) 2. One to two years of experience 3. Procedures and patient case experience - appropriate for the specialty (attestation of knowledge and skills). 4. A specialty exam. 120 Questions from specialty Costs $350 This is an additional exam from your PANCE or PANRE

What is mandatory after 2020 for EDUCATION? Current (Since 2010) & The Future EDUCATION

Masters Degree Mandatory after 2020 except Military programs accredited prior to 1/1/06 are not required to confer graduates of the PA program a baccalaureate or graduate degree Ask

What progam is administered by state?

Medicaid

◦Identify the allied health professional below: Need Lab results

Medical Technologist

◦Identify the allied health professional below: Need Type and Crossmatch for 2 units of blood

Medical Technologist

Where to register PANCE? How to get NYS license and how long does it valid for?

NCCPA }Certificate of Professional Education Completed by program director Requires official transcript that is complete }NCCPA examination score requests and submission is your responsibility - NYS License valid for 3 years

T/f, you should use alcohol sanitizier if your hand is visibly soiled?

NO! Use Soap and water instead.

Is DNI the same as DNR?

NO!!! ◦Do not resuscitate order prevents medical professionals not to perform cardiopulmonary resuscitation (CPR)

Can private insurance be part of Shared/Split Visits or Incidence?

NOPE only medicare

Is Self Assessment and PI CMEs mandatory?

NOT MANDATORY ANYMORE }Self Assessment and PI CMEs Started from 2014 cycle by NCCPA Hard to find Very Expensive Difficult for PAs to complete Resisted by the AAPA No longer a requirement as of August 2016

What is patient autonomy?

Obligation to respect patients as individuals and to honor their preferences in medical care.

When did the first PA students come to be?

October 1965 - First Four students, all ex-Navy entered the 2-year program at Duke (3 students graduated) Philosophy included: -education and orientation similar to physician - focus on assisting family practice physicians and internists - lifelong learning skills with focus on on-going growth

When was first PAs graduated from Duke university?

October 6, 1967

What is Specialty Certificates of Added Qualifications (CAQs)?

Offered by NCCPA Achieve recognition for your specialty experience, skills and knowledge Specialties: ◦Cardiovascular/Thoracic Surgery ◦Emergency Med ◦Hospital Medicine ◦Nephrology ◦Orthopedic ◦Pediatrics ◦Psychiatry

What does outpatient setting mean?

PAs in Private Practice & Outpatient Settings ◦Solo or group practice ◦Multispecialty practice ◦Urgent care ◦Ambulatory surgical centers ◦Corrections institutions ◦Community health centers ◦School and university health centers ◦Mental health facilities ◦Hospital-based clinics

What does the NYS State Education Law (Article 131B) state?

PAs may provide medical services only under the supervision of a licensed physician (MD or DO) Delegation of duties remains with the supervising physician

What does the state education law currently say about whom to assign the PA role?

PAs may provide medical services only under the supervision of a licensed physician (MD or DO) These physicians will delegate what we can or can not do.

Which model is used for quality improvement?

PDSA model - plan, do, study, act

What are two fund from medicare?

Paid through 2 trust fund accounts held by the US Treasury ◦Hospital Insurance Trust fund ◦Supplementary Medical Insurance Trust Fund

How would you enroll in part A/B or part D?

Part A is automatically enrolled when > 65 if requirement meets. Part B is enrolled with payment of premium Part D requires only through private insurance companies that have contracts with the government. ~ only when meet requirement.

When can confidentiality be breached?

Patient confidentiality is not absolute and can be breached if there is a conflict between ◦Patient autonomy (the right of the patient to control his or her own health information) ◦ ◦Non-maleficence (protecting the patient or others from harm)

What are some issue pt may have regarding to PA?

Patients: ◦Confuses our roles with Physicians ◦Roles compared to Nurse Practitioners and Nurses

What are penalties of Medicare Fraud?

Penalties ◦Fines as high as $10,000 per offense ◦The provider may be banned from participating in Medicare ◦Up to five years in prison per offense ◦Revocation of licensure Administrative Sanctioning ◦If fraud is the result of bad administration ◦Same penalties may apply!!!

What are types of Medicare Fraud?

Phantom Billing - never happened: ◦The medical provider bills Medicare for: Unnecessary procedures, or procedures that are never performed Unnecessary medical tests or tests never performed Unnecessary equipment; or equipment that is billed as new but is, in fact, used. Patient Billing - [patient agrees to say yes in exchange to benefits]: ◦A patient who is in on the scam provides his or her Medicare number in exchange for kickbacks. ◦The provider bills Medicare for any reason and the patient is told to admit that he or she indeed received the medical treatment. Upcoding scheme and unbundling: ◦Inflating bills by using a billing code that indicates =The patient needs expensive procedures =A more detailed visit was performed (when indeed it was not)

When you have a patient presenting with UTI symptom, rather than doing UA test, you billed the UA test, what type of medicare fraud is that?

Phantom bills

What is PAEA?

Physician Assistant Education Association The only national organization representing Physician Assistant educational programs Focus is on the educational programs, started combined with AAPA in 1972, formerly known as APAP, work with AAPA. The new name in 2006 when they became independent APAP - 1972 -> 2005 PAEA - 2006 - now

the Association of Physician Assistant Program (APAP) is now known as ___________________.

Physician Assistant Education Association (PAEA)

what does AAPA believe about ways to enhance the delivery of high quality health care?

Physician and PA team relationship is fundamental.

What are the key aspect of social influences in The 2000s - Moving forward?

Powerful national and international events; 9/11 attack, war in Iraq, terror attack in Europe }Increase demands for the healthcare professional to care for the injured and wounded }Baby boomers about to retire ◦That born post-WWII (1946-1964) The older generation of healthcare workers are about to retire

What is the following example considered as? Patient had wrong limb amputated unavoidable adverse event? Error of omission? Negligence? Ameliorable adverse event? Preventable Adverse events?

Preventable Adverse events

What is the following example considered as? Patient with known allergy to specific meds, receive the med and develop reaction unavoidable adverse event? Error of omission? Negligence? Ameliorable adverse event? Preventable Adverse events?

Preventable Adverse events

What is the definition of Preventable Adverse events?

Preventable Adverse events - an adverse event injury that could have been avoided as a result of an error or system design flaw. Ex - Bed check for patients with an unsteady gait

Got a job? Got Hired? What Next in private practice?

Private Practice ◦Process is quick and easy Usually one person makes the decision Verify references, call previous employers Can usually start working the same week ◦The next step is obtaining malpractice insurance ***(paid for by yourself or by practice) ◦Electronic Medical Record training ◦Any other facility-related training

In NYS, how many PAs can one physician supervise? Private office Correctional Facility Licensed facility

Private office - no more than 4 PAs Correctional Facility - no more than 6 PAs Licensed facility (Hospitals/LTC, etc) - no more than 6 PAs

What are included in a written discharge plan? Anything not required here? A Procedures B Pertinent results C Discharge medications D Consultation note E Follow up

Procedures Pertinent results Discharge medications Follow up NO CONSULTATION NOTE

What is medical professional's explanation of professionalism?

Professional conduct is a belief system in which group members ("professionals") declare ("profess") to each other and the public the shared competency standards and ethical values they promise to uphold in their work and what the public and individual patients can and should expect from medical professionals."

ARC-PA philosophy

Programs should focus on high-quality PA education and should have a continuous self-study process (decide what is wrong and how to fix it via surveys, etc) - Program self evaluates and ensures that the quality is there.

What is the privacy rule?

Protected Health Information The Privacy Rule protects all "individually identifiable health information" in any form including oral, written or electronic. - all forms !! REGULATED in a federal level.

What can PA provide across all specialties?

Provide medical care and care coordination Increase patient access Contribute to improved quality Are a cost-effective resource for meeting patients' medical needs.

What provider might need to provide to the patient when conducting disclosure of medical errors? (2)

Providers will need: ◦Disclosure coaching ◦To be informational, instrumental(serving as a means of pursuing an aim or policy.) and give emotional support

What is Palliative Care?

Treatment that enhances comfort and improves the quality of an individual's life who is facing a serious illness but may not qualify for hospice care [expected to live a little longer]. •The expected outcome is relief from distressing symptoms, the easing of pain, and/or enhancing the quality of life.

T/F Most Insurance Companies offer one or more of the following types of plans: }HMOs - Health Maintenance Organization }PPOs - Preferred Provider Organization }EPOs - Exclusive Provider Organization }POS plans - Point of Service }CDHPs (Consumer Directed Health plan or HDHP (High Deductible Health Plan)

True

T/F students must assume everyone may be able to transmit potential infectious diseases

True Universal Precautions is Infection control guidelines designed to protect health care providers from exposure to diseases spread by blood and certain body fluids

T/F - health plan do need to comply with HIPPA regulation?

True! - The ACA has established industry-wide standards for use by clinicians of electronic systems to determine patient eligibility for health coverage and check on the status of health claims. [◦Example - you visit your PCP and they check your Insurance eligibility electronically] - Health plans must comply with HIPAA regulations.

What are the mobile Device Privacy and Security? T/F, EPHI is covered under HIPAA regulation.

True! Electronic Protected Health Information (EPHI) is covered under HIPAA regulations. Things considered electronic: Laptops, personal computers, PDA's, Smart phones, public workstations, USB flashdrives, discs, CD's, e-mail, remote access devices can be vulnerable.

T/F, clinician using mobile device to access patient information may violate privacy regulation?

True, base on electronic protected health information

What is the character that is central to therapeutic relationship between clinicians and patients?

Trust

What are two main types of codes?

Two Main Types of Codes }Diagnostic Codes ◦ICD-10 (International Classification of Diseases) }Procedural Codes ◦HCPCS (Healthcare Common Procedure Coding System) ◦CPT (Current Procedural Terminology) is a sub-set of HCPCS codes

What are typical Duties include in inpatient setting?

Typical Duties include: ◦Admit new patient ◦Inpatient rounds ◦Discharge summaries ◦Perform clinical procedures ◦Order and interpret medical tests ◦Orders for medications and ongoing treatment ◦Documentation - H&Ps, progress notes, consult notes, SOAP notes, procedure notes - appropriate for billing ◦Involved in surgical procedures ◦Work as "house officers" ◦Work various shifts - 8, 12, 16 hours ◦Other duties directed by the facility and supervising physician

What is ICE?

U.S. Immigration and Customs Enforcement

What is the limitation to New York's Health Care Proxy law in terms of appointing health care proxy?

Under New York's Health Care Proxy law, you can appoint any competent adult as your Health Care Agent except: ◦You cannot appoint your doctor as your health care agent unless your doctor is your spouse or your relative. ◦ ◦You cannot appoint an operator, administrator, or employee of the hospital or nursing home where you are admitted unless they are a relative or you appointed them before your admission.

How to Become a leader in health care?

Understand what is done How is it done What do you produce? Keeping authentic and understand the situation

Which of the preventable medical errors contribute to the majority of medical error? }Medication errors }Under or over treatment }Clinician burnout }Misdiagnosis }Lack of education or experience }Surgical errors }Incompetent staff }Poor communication

}Medication errors

What are some causes of Preventable Medical Errors? (8)

}Medication errors }Under or over treatment }Misdiagnosis }Lack of education or experience }Surgical errors }Incompetent staff }Poor communication (eg. illegible handwriting, language barriers, inaccurate documentation, poor team work) }Clinician burnout (fatigue)

What are MODIFIER TO CPT CODE?

}Modifier - is used to indicate that another service/procedure was performed on the same patient on the same day, that was separate and distinct from the original evaluation and management (E/M) code used.term-41 Doing something else other than reason came in for ◦Must have clear and proper documentation ◦Different modifiers for different types of services Most Modifiers are procedure-related not ICD.

What if you failed PANRE 4 times?

}NCCPA will work with you to complete CME requirements }Or Opt in for the "Alternative to PANRE" option

What medicaid program covers preganant woman?

}NYS Children's Medicaid Program ◦Also covers pregnant women

What is the difference between living will and health care proxy?

}NYS has no statute governing Living Wills. The provider does not have to abide by that but has to abide health care proxy. Court of Appeals has established they can be valid, but it: ◦Need to express "clear and convincing" evidence of the patient's wishes }Serves as a guideline ONLY for medical practitioners to provide care }Needs two witnesses signatures

What are Private Insurance Reimbursement for PAs?

}Nearly all private payers cover medical and surgical services provided by PAs. }Often payers will defer to state law supervision requirements even when the PA's services are billed under the name and NPI number of the physician }Some payers may separate PA/Physician credentials ◦However, payment for services provided by PAs is typically made regardless of whether payers separately credential PAs. }Private health insurance companies do not necessarily follow Medicare's coverage policy rules

What is network model of HMOs?

}Network Model ◦Combination of Group Model and Open-Panel Model ◦HMO contracts with both multi-specialty practices and IPA's Patients have more choices of physicians ◦Most popular of recently established HMO's }Often HMOs used mixed models to deliver healthcare

What component Determine the Level of Service needed in New pt and established ppt?

}New Pts ◦The history, exam and medical decision-making need to be at the same level or higher of complexity }Established patient ◦Two of the three components must be at the same level or higher to support the level of care. ---- }The history and/or exam must be appropriate to the patient's presenting problem ◦Therefore, medical decision-making always will be one of the components for an established patient encounter.

}New Pts with higher level code

}New Pts 9920_ ◦The history, exam and medical decision-making need to be at the same level or higher of complexity

What is Affordable Care Act Goals?

}Nicknamed "Obamacare" }Signed into law March 23, 2010 Goals ◦Increase access to affordable and quality health insurance ◦Curb the growth of health care spending ◦Lower the uninsured rate ◦Allow insurance for all regardless of sex or pre-existing conditions Affordable Care Act is an Expansion of Medicaid eligibility

What is New in the PA World?

}OTP }Name Change }Hospice }PA Student Documentation

What is OPTIMAL TEAM PRACTICE (OTP)?

}OTP - a concept designed to eliminate state-mandated supervisory requirements that may be impeding PA employability and practice in some states }AAPA's House of Delegates unanimously approved a new policy in support of "Optimal Team Practice (OTP) - May 18, 2017 }Previously named "Full Practice Authority for PAs (FPAR). ◦Renamed because AAPA want to emphasize the team/collaborative practice

What is OTP?

}OTP calls for laws and regulations that: ◦Emphasize PAs' commitment to team practice Degree of collaboration determined at practice level ◦Eliminate legal requirements for supervision in order to practice ◦Authorized PAs to be directly reimbursed ◦Establishment of autonomous state boards to license, regulate and discipline PAs (or for PAs to be full voting members of medical boards)

99213 falls where?

}Office Visits ◦99201 to 99205 are for new patients ◦99211 to 99215 are for follow-up (Established) visits

E/M CPT codes 992**___** /_

}Office Visits ◦99201 to 99205 are for new patients ◦99211 to 99215 are for follow-up (Established) visits }Hospital visits ◦99221-> 99223 - New patients ◦99231-> 99233 - Established patients }Outpatient Consultation ◦99241-> 99245 }Inpatient Consultation ◦99251->99255 }Emergency Dept ◦99281 -> 99285 Out/In/Out/In ~> 8 [infinity sounds like Emergency ]

How long is CAQ valid for?

}Once awarded, a CAQ will be valid for a period of ten years (provided that licensure status and PA-C certification are maintained).

What are in the optional section of heath care proxy?

}Optional Sections ◦Limitations on agents authority Artificial nutrition ◦An alternative agent ◦Wishes with regard to organ donation ◦Expiration date or situations that will trigger expiration of the proxy Proxy will remain in effect until a new one is written

Which is this statement describing? ◦Provider bill patient directly }Out-of-Pocket }Private Insurance }Public Insurance

}Out-of-Pocket

Where are three places you can bill as PAs?

}Out-of-Pocket ◦Provider bill patient directly }Private Insurance ◦Provider Reimbursed by private Insurance ◦Copays and deductibles may apply }Public Insurance ◦Providers bill public plans directly (straight Medicare and Medicaid) ◦Or bill Managed Care Organizations (MCOs) - if contracted with the government

What are some other names for PA?

}PA often referred to as: ◦Mid-Level Provider (MLP) ◦Advanced Practice Provider (APP) ◦Non-Physician Provider

What are the different types of relationships PA can have?

}PA-Physician relationship }PA-Nursing relationship }PA-Allied Health relationships }PA-Support staff relationship

NCCPA Examinations

}PANCE ◦Initial Certification }PANRE ◦Recertification Exam

When can you take PANRE?

}PANRE Taken in 9th or 10th year }Once every 90 days }Up to two (2) times in the 9th year }Up to three (3) times in the 10th year }Maximum of four (4) TOTAL attempts

}Describe the PA-Support Staff relationships

}PAs - work as a team with support staff

What are the Professional Behavior PA should have toward public?

}PAs are expected to demonstrate responsiveness to the needs of patients and society }PAs are expected to demonstrate a commitment to ethical principles pertaining to provision or withholding of - clinical care - the confidentiality of patient information - informed consent - business practices }PAs are expected to demonstrate accountability to patients, society, and the profession }PAs must demonstrate adherence to legal and regulatory requirements, including the appropriate role of the PA

What are the Professional Behavior PA should have towards the Patient?

}PAs must prioritize the interests of those being served above their own }PAs must demonstrate a high level of ethical practice }PAs must demonstrate a high level of sensitivity and responsiveness to a diverse patient population, including culture, age, gender, and disabilities }PAs are expected to demonstrate respect, compassion, and integrity

NYS Prescription

}Paper prescription are rarely used due to Electronic Medical Records }Website: http://www.health.ny.gov/professionals/narcotic/official_prescription_program/order/ }Register using form DOH-4329 }Order scrips using form DOH-250 }Can order prescription online by registering at the Health Commerce System https://commerce.health.state.ny.us/public/hcs_login.html }Note: Prescriptions will be shipped to the address on your DEA registration }Information for your supervising physician will be imprinted on your scripts

what are the principles of interprofessional collaboration? <7>

}Patient & family centered }Community & population oriented }Relationship focused - Team Based }Communication focused }Outcomes-driven }Applicable across professions }Sensitive to systems Based practices }

Which of the following describes the term? ◦Meeting patients' needs and preferences and providing education and support. A. Effectiveness B. Efficiency C. Equity D. Patient centeredness E. Safety F. Timeliness

}Patient centeredness. ◦Meeting patients' needs and preferences and providing education and support.

Which this following statement describing? ◦Prohibits a physician from making a referral for certain designated health services to an entity in which the physician (or member of his or her immediate family) has an ownership/investment interest or with which he or she has a compensation arrangement ◦False Claims Act ◦Anti-Kickback Statute ◦Physician Self-Referral Law (Stark Law)

}Physician Self-Referral Law (Stark Law) ◦Example: A provider refers a beneficiary for a designated health service to a business in which the provider has an investment interest Ex: Lab, Radiology, OT, PT, Pharmacy services, etc

Shared/Split Visits T/F Physician does not have to examine and document face-to-face involvement with patients.

}Physician must examine and document face-to-face involvement. }Must contain legible signatures and credentials of both providers

What are the Goals of IPC/IPE? <2>

}Prepare all healthcare professions students for deliberately working together with the common goal of building a safer and better healthcare system. }Improved team-based training (knowledge, skills, attitude) that leads to improved quality and safety in team-based patient care (behaviors, competence).

Which is this statement describing? ◦Provider Reimbursed by private Insurance ◦Copays and deductibles may apply }Out-of-Pocket }Private Insurance }Public Insurance

}Private Insurance

What are Various Forms of Health Care Insurance?

}Private Insurance ◦Managed Care Organizations }Government Insurance ◦Medicare ~ administered by federal ◦Medicaid ~ administered by state

What are four places PA can be in at Healthcare Settings?

}Private Practice & Outpatient Settings }In-Hospital Settings }Long-Term Care Settings }Home Visits

What are Malpractice Insurance?

}Usually provided by the facility if not ask for higher salary for compensation. }Determined during negotiation }Limits of liability coverage ◦Should have enough coverage in the event of an incident ◦Usually depends on your practice Higher risk - eg. Cardiovascular Sx, Neuro Sx, Ob/Gyn - require higher coverage Lower risk - eg. Primary care, internal med - lower coverage will suffice ◦If being paid by a facility - coverage amount will be determined by them

What are unacceptable Physician Documentations?

}Unacceptable Physician Documentations ◦"Patient seen and evaluated" ◦"Seen and Agree" ◦"Agree with above" ◦"Discussed with PA/NP. Agreed" ◦"Agree with PA/NP"

What are three categories of quality problems?

}Underuse: Many patients do not receive medically necessary care. }Misuse or abused: Each year, >100,000 Americans get the wrong care and are injured as a result. Each year, > 1.5 million medication errors }Overuse: Many patients receive care that is not needed or for which there is an equally effective alternative that costs less money or causes fewer side effects.

How can effective Communication be achived? [4]

}Verbal }Non-Verbal communication: written, gestures, etc }Information in various languages }Interpreter services }Effective communication between patients, their families and other healthcare providers improves patients' outcomes

What were AAPA formerly known as?

◦American Association of Physician's Assistants (1968-1971) ◦American Academy of Physician's Associates (1971-1972) ◦American Academy of Physician's Assistants (1972-1980) ◦American Academy of Physician Assistants (1980) Apostrophe stayed till 1980*

According to health disparity, everyone deserve what?

◦An equal opportunity to make the choices that lead to good health ◦Access to affordable and quality healthcare

What is hospital DNR?

◦Issued if you are in a health care facility such as a hospital, nursing home, or a mental hygiene facility licensed by New York State - Emergency personnel (EMS), such as ambulance paramedics must honor your Hospital DNR Order during transfer

leadership style: situational or Contingency Leadership Models

◦Leadership is based on the position of power, the interactions between the leader and members, and the desired goal - Eg. Running a code

What may be consequences of medical error to you?

◦Legal or institutional retaliation Working with your institution Malpractice concerns ◦Professional concerns Colleagues will lose trust Patients will lose trust

What is advantages of medicare advantages plans of part C?

◦Many plans cost $0 month - if you also qualify for Medicaid. ◦Usually includes drug coverage [one card cover everything] ◦May include hearing and dental plans ◦Can include gym discounts ◦Can switch to another Managed Care plan during open enrollment

What is AHRQ's definition of quality measure?

◦Mechanism to assign a quantity to quality of care

What are the features of a good PA-Physician relationship? 5 MUST HAVE! My Honey Was Always Positive!!!

◦Mutual Understanding ◦Honesty and Respect ◦Willingness to teach and learn (both parties) ◦Availability ◦Patience My Honey Was Always Positive!!!

What are the 5 federal service areas included in the chapter organizations?

◦Naval association of PAs ◦Public Health Service Academy of PAs ◦Society of Air Force PAs ◦Society of Army PAs ◦Veteran Affairs PA Association

When would a living will be valid?

◦Need to express "clear and convincing" evidence of the patient's wishes.

Common vulnerabilities among the immigrant population How does residential Location play a role in immigrants?

◦New residence/new location - less likely to have culturally competent providers or other support from community-based organizations ◦Immigrants utilize the Emergency room for emergencies.

What is one element of six that NYS does not met?

◦No restriction on the number of PAs with whom a physician may supervise

What is non-hospital DNR?

◦Non-Hospital DNR ~ Must be recorded on a state specific form [DOH- 3474] - for outpatient.

What is the following statement describing? ◦Allows physicians to see patients outside of the HMO's network with referrals ◦HMO contracts as an independent practice association (IPA) HMO pays on a capitation basis ◦Out of network referrals often do not have as much coverage ◦Staff Model ◦Group Model ◦Open-panel Model ◦Network Model

◦Open-panel Model

What is coverage of Medicare Part D?

◦Outpatient prescription drug coverage - routine daily drugs ◦Provided only through private insurance companies that have contracts with the government ◦It is never provided directly by the government (like Original Medicare is).

What are changes in PA as of Chapter 48 of the Laws of 2012?

◦PA now can sign death certificate. ◦Separates Physician Assistant and Special Assistant in the law

What can PAs do in surgical practices? [4]

◦PA perform: Pre-op H&Ps, order and interpret labs, first assist, post-op care [4] . ◦Both PAs and Physician round on patients (together or separate)

NYS: PA Legislation & Regulations Miscellaneous

◦PAs are covered under the "Good Samaritan Act" provided that compensation has not been given for services rendered ◦PAs are not excluded from jury duty ◦In NYS, PAs are mandated child abuse and maltreatment reporters ◦PAs are required to complete infection control practices training (every 4 years) ◦PAs are authorized to certify a disability for the purpose of obtaining a disability parking permit ◦Group 6 to a summary of all recent updates (<2 years) in PA Laws and practice

What are the benefits of PA and Physicians working together in general practice?

◦PAs can manage their own patients in general practice. ◦PA-Physician work together as a team: Increase volume Some patients seen alone by PA Some patients seen together with physician (Complicated cases) Reinforces continuity (when usual provider not available)

What are in the second section of NYS patient right?

◦Patient Bill of Rights The second section of the document explains the laws relating to what services a hospital must provide patients Things that facility must be provided to patient

What is capitation?

◦Payment for services per-member per-month ◦The physician is paid the same amount per member regardless of the number of visits member has.

Who can have a DEA number?

◦Physician ◦Physician Assistant ◦Nurse Practitioner ◦Pharmacist ◦Dentist

What is a packrat?

◦Physician Assistant Clinical Knowledge Rating and Assessment Tool (PACKRAT)

What were the early intervention of student assessment?

◦Physician Assistant Clinical Knowledge Rating and Assessment Tool (PACKRAT) There are two we will complete in York college, - didactic PACKRAT ~ everything we covered [PA prof, I & C, and E-port] - clinical PACKRAT

Which is this following statement describing? Which law is in place preventing it? Sees PCP and down the block, there is a PT service. Everyone with MS issue gets PT service. ◦False Claims Act ◦Anti-Kickback Statute ◦Physician Self-Referral Law (Stark Law)

◦Physician Self-Referral Law (Stark Law)

What is Group model of HMOs?

◦Physicians and medical professionals are not hired directly. - They belong to multi-specialty physician group practices ◦HMO pays the group in bulk, and the physicians decide within the group how the money is distributed ◦Physicians only see patients that signed up for the HMO that contracted them

What are some concept may hinder PA - Physician Relationship?

◦Physicians may not fully understand our roles and capabilities

What is health Insurance?

◦Pre-paid program that reduce the cost of medical expenses for individuals

What is difference between QI and QA?

◦QI relies on data-driven decision making ◦QI improve processes and systems

What are the five rights of medication administration?

◦Right medication ◦Right patient ◦Right dosage ◦Right route ◦Right time (frequency) Verify patient's medication - from patient, pharmacies, other clinicians, family members }Use weight-based dosage - especially for kids }Use appropriate patient identifiers }Clear and detailed documentation }Verify allergies and reactions }Medication Reconciliation }Utilize Pharmacists }Be familiar with your institution's medication administration policies, regulations, and guidelines. }Have a drug guide available (PDR or electronic format - Epocrates)

What is Open-Panel model of HMOs?

◦Similar to Group Model ◦ ◦HMO contracts as an independent practice association (IPA) IPA - An association of independent physicians Can be for-profit HMO pays IPA on capitation basis -> IPA then pays its members ◦Allows physicians to see patients outside of the HMO's network with referrals ◦Out of network referrals often do not have as much coverage

What is disadvantages of medicare advantages plans of part C?

◦Still requires individual to pay copays and coinsurance fees ◦Plans are difficult to compare between different plans ◦Usually, no nationwide coverage - Only work in the state you are in. [medicare has nationwide coverage] ◦Requires referrals to specialist ◦Plan can change medical and drug benefits every year

If you decide to go forward with a disclosure, what should you do? In which order?

◦Talk to the patient first ◦Ask for the patient's consent. ◦Ideally, the patient will consent to the disclosure. ◦If not, it is still okay to reveal the information, if you determined that it is legal and ethical to do so. DO it in a way that minimize patient harm.

When can people to make informed health care decision?

◦The ability to understand and appreciate the nature and consequences of health care ◦The ability to reach an informed decision

leadership style: involving followers in the goal.

◦The leader gets others to buy into her vision for the organization, and to make it their own. ◦She may accomplish this through charisma, through the force of her own belief in the power and rightness of the vision, or through the nature of the vision itself.

What is premium?

◦The money paid to a health plan on a regular basis for coverage

What is fee for service?

◦The patient sees a provider, then the provider bills the health plan per professional activity - (unbundled services) Physical examination Suturing unbundled services - insurance pay per service provided.

What is co-payment?

◦The portion of a claim or medical expense that a member must pay out-of-pocket. Usually a fixed amount

What is the deductible?

◦The portion of a health plan subscribers health care expense that must be paid out-of-pocket before any insurance coverage applies. ◦ ◦May apply to only a portion of the plan - example Pharmacy services Durable medical equipment (DME) Walkers Glucometers Home hospital beds

What is a structure in the measurement of 6 IOM quality domains evaluation?

◦The resources and organizational arrangements in place to deliver care. }Examples: ◦Number of nurses per patient. ◦Percentage of physicians who are board certified. ◦Presence of quality improvement programs.

How many supervising provider can a PA have?

◦There is no limitation on the number of supervising physician that may be assigned to one PA ◦Supervision must be ongoing and continuous, but not "direct" (except under limited permit)

What is your job if your patient give you the advanced directive?

◦Under New York law, any doctor who is given a Health Care Proxy form must arrange for the form, or a copy of the form, to be put in your medical record.

Common vulnerabilities among the immigrant population What is welfare reform?

◦Undocumented immigrants has no access to healthcare services excepts children - eg. CHIP program covers them

In what forms can student experience mistreatment? [6]

◦Verbal ◦Sexual ◦Physical ◦Gender-Based ◦Race-Oriented ◦Religious-Based

What are the general protocol for exposure? (5 steps)

◦Wash area with soap and water ◦Use eye splash station ◦Notify supervisor, go to ER or employee health ASAP (don't wait for end of shift) ◦Get name of patient ◦Patient will consented to be tested for: HIV, hep B, hep C

What safety policies do PA students need to know for clinical rotations?

◦What to do if you sustain an injury ◦Requirements for male & female exams (chaperones) ◦What to do in an emergency ◦Who have access to clinical spaces

What does it mean by "how to get information and assistance"?

◦Who to ask questions ◦Who will arrange special help ◦Rights of privacy and family contact ◦Where and how to obtain interpretation Also contains a Glossary to help understand terms commonly used in the hospital

Questions: ◦Legal status, access to social services, jobs with benefits, healthcare coverage (lower rates) ◦socioeconomic background ◦immigration status ◦limited English proficiency ◦federal, state, and local policies on access to publicly funded health care ◦residential location - limited resources ◦Stigma and marginalization

◦immigration status

What is Protected Health Information?

◦information about health status ◦Provisions of health care to an individual (i.e: any healthcare services provided to the patient) ◦Payment for health care that can be linked to a specific individual }Includes any part of a patient's medical record or payment history.

Questions: ◦Healthcare provider does not speak their language ◦socioeconomic background ◦immigration status ◦limited English proficiency ◦federal, state, and local policies on access to publicly funded health care ◦residential location - limited resources ◦Stigma and marginalization

◦limited English proficiency

}To improve individual health and build healthy communities, health care providers need to what?

◦recognize and address the needs of - the unique culture - language and health literacy of diverse consumers and communities.

Questions: immigrants, in general Are less likely to graduate from high school and more likely to occupy service occupations ◦socioeconomic background ◦immigration status ◦limited English proficiency ◦federal, state, and local policies on access to publicly funded health care ◦residential location - limited resources ◦Stigma and marginalization

◦socioeconomic background

What is the statement following describing? This document allows you to appoint someone you trust to make health care decisions for you if you lose that ability. - You can include an alternate - if the primary agent is unavailable - Also contains information on organ donation wishes - Everyone over the age of 18 is encouraged to appoint an agent - in the event of Temporary/Permanent inability to make health care decisions A. ◦Health Care Proxy B. ◦Living Will C. ◦Living will with Health Care Proxy D. ◦DNR

A. ◦Health Care Proxy A health care proxy can be - ◦Family Member ◦Friends

What is interprofessional practice?

A philosophy of patient care

What is the living will?

A written declaration of health care wishes describe situations where you would or wouldn't want care ◦The Living Will is intended to anticipate the situation wherein you might be in an incurable or an irreversible mental or physical condition, with no reasonable expectation of recovery ◦Example Artificial nutrition Mechanical ventilation Hemodialysis Blood transfusions

What year did Dr. Charles Hudson introduce the concept of a Physician Assistant and Mid-Level Provider emerged A. 1961 B. 1965 C. 1964 D. 1970

A. 1961

How long was the formal training for barefoot doctors? A. 2-3 months B. 2 years C. 1 year D. 4 months

A. 2-3 months

What does the following statement entail? ◦Payment for services per-member per-month - ◦The physician is paid the same amount per member regardless of the number of visits member has. A. Capitation B. Fee for service C. Explanation of benefit D. Network

A. Capitation

Who created the MEDEX model? A. Dr. Richard Smith B. Dr. Henry Silver C. Dr. Charles Hudson D. Dr. Amos Johnson E. Dr. Eugene Stead

A. Dr. Richard Smith

Who wanted to avoid the terms "assistant" or "associate" as they were potentially demeaning. He favored "Medex"? A. Dr. Richard Smith B. Dr. Henry Silver C. Dr. Charles Hudson D. Dr. Amos Johnson E. Dr. Eugene Stead

A. Dr. Richard Smith

What are the six domains of quality in institute of medicine?

A. Effectiveness B. Efficiency C. Equity D. Patient centeredness E. Safety F. Timeliness

What is the following statement describing? - An organization that provides or arranges managed care for a health insurance company - A health plan that utilizes primary care physicians as gatekeepers - Based on four models ◦Staff Model ◦Group Model ◦Open-panel Model ◦Network Model - Uses capitation or fee-for-service depending on plan enrolled A. HMOs - Health Maintenance Organization B. PPOs - Preferred Provider Organization C. EPOs - Exclusive Provider Organization D. POS plans - Point of Service E. CDHPs (Consumer Directed Health plan or HDHP (High Deductible Health Plan)

A. HMOs - Health Maintenance Organization ◦HIP Health Plan of New York is the largest HMO in New York City.

Where is "Rights of privacy and family contact" located in NYS Patient right? A. ◦Information on how to best get assistance B. ◦Explains patient rights

A. ◦Information on how to best get assistance

What is an established patient?

An established patient is one who has been seen by the physician, or if in a group practice, by a physician in the same specialty within the past 3 years.

What are 10 Difficult Employees? Not for exam

An important part of dealing with difficult people is to understand where they are coming from. Generational differences can have a impact on communication. MANAGING PROBLEM EMPLOYEES AND DIFFICULT PEOPLE THE UNDECIDER He takes days to make a decision and then, after it's made, revisits it. Then revisits it again. Then, when things fall apart and he is held responsible for his indecision, he becomes indignant or evasive. "It's not MY fault!" How to cope: Establish a deadline where the decision must be final, and a default decision that will hold true if no decision is made. When the deadline comes, that's it. Refuse to consider any other alternatives. THE ULTRA-COMPETITOR No matter how a situation plays out, no matter who gets hurt in the process, the ultra-competitor can't let it go until he's convinced that he's won--and, more importantly, that someone else has lost. How to cope: Get him focused on having the entire team win, rather than just him. Hint: Pay him a bonus based on team achievement--never on individual accomplishment. THE DRAMA QUEEN He automatically turns absolutely everything into a hissy fit, replete with pique, umbrage, and a host of other French emotions. He seems to draw energy from the drama while draining energy from everyone else. How to cope: Set up boundaries for the behavior that you won't tolerate. Eject him from any meeting where his behavior becomes obstructive. THE ICONOCLAST He thrives on the negative attention that comes from dissing authority figures and social protocols. He misses deadlines just to prove he doesn't have to follow the rules and takes up causes without really understanding the implications of his actions. How to cope: It's all about aiming him at the right enemy. Oddly, these types often do well as "customer advocates" who can take on the bureaucracy in order to see that customers get what they need. THE DRONER He's always ready to give you a presentation--and it's usually one you've heard before. He's got a list of bullet points and is going to read each and everyone to you, or know the reason why! How to cope: Have a written agenda for every meeting, with a limited amount of time for presentations. Better yet, make a "no PowerPoint" rule for your meetings. Then stick to it. THE SOCIAL (NETWORK) BUTTERFLY He is convinced that it's productive for him to remain online all day "building relationships" with all your customers. In fact, he's just adding to the day-to-day blather that's such an integral part of the social network. How to cope: Assign him measurable goals--like a certain number of qualified sales leads that he has to create every week. THE VOLCANO He explodes whenever things don't go the way he thinks they should. He screams at meetings, yells into the telephone, and gets in your face. While he might apologize later, the whole team ends up perpetually walking on eggshells. How to cope: Raise your own intensity (or you won't be heard), and then refuse to put up with unprofessional behavior. If necessary, leave the room until he's cooled down. THE PROCRASTINATOR He says yes to projects but fails to follow through. As deadlines approach, he can't be found, even via email. When the work is finally turned in (often by others who have covered for him), he'll go on a mini-vacation to "recuperate from the stress." How to cope: Unfortunately, the only solution here is a little good old-fashioned micromanagement. Layout frequent (even daily) milestones, and create consequences for missing one -- or for failing to report that he missed it. THE CREATIVE GENIUS He's a legend in his own mind ... and makes certain that you know about it. He's always talking about the amazing stuff he did in the past and his equally amazing plans for the future. Still, he seldom seems to actually do anything today. How to cope: Give some lip service to his greatness, then bring him down to earth by breaking a project into chunks and getting him to "consult" on each chunk. THE PANIC BUTTON Some people really shine in a crisis. Others ... not so much. This guy remains calm for days and weeks, but then when a problem has reached its inevitable conclusion, he runs around like a decapitated chicken. How to cope: Create an early warning system so that there are fewer surprises. And replace the regular coffee with the decaf on the day the bad news hits.

Which agency is the federal agency that responsible for improving the quality of health care for all Americans?

Agency for Healthcare Research and Quality (AHRQ)

When can a agent decision be reversed?

Agent's decision is final unless an objecting family member or facility obtains a court order overriding the decision or disqualifying the agent.

Medicaid

Aid the poor

What benefits from state laws defining PA-Physician collaboration?

Allows for customization of healthcare teams Best meet the needs of the patients

How did informal PA training emerge?

Also occurred around 1961 *Collaboration begins Dr. Amos Johnson publicized his training of Mr. Henry Lee "Buddy" Treadwell in 1967 - Family practice in North Carolina - Buddy started off as a receptionist and worked his way up to do most procedures in fam medicine except delivering babies

What is the definition of Ameliorable adverse event?

Ameliorable adverse event - an injury whose severity could have been substantially reduced if different actions or procedures had been performed or followed. Ex - did not provide early Abx for sepsis pts.

Who and when was AAPA funded and where?

American Academy of PAs Founded in 1968 by first students/grads of Duke Program Represents over 108,500 PAs across all specialties NORTH CAROLINA

Who are members of Members of ARC-PA?

American Academy of Physician Assistants Physician Assistant Education Association American Medical Association }American Academy of Family Physicians }American Academy of Pediatrics }American College of Physicians }American College of Surgeons

What is GoodRx?

An American healthcare company that operates a telemedicine platform and a free-to-use website and mobile app that track prescription drug prices in the United States and provide free drug coupons for discounts on medications.

What is Medicare Advantage Plans?

Combine of medicaid and medicare Medicare Advantage Plans is private Managed Care plans that contract with Medicare to provide services ◦Health Maintenance Organization (HMO) ◦Preferred Provider Organization (PPO) ◦Private Fee-for-Service (PFFS) ◦Special Needs Plans (SNPs) ◦HMO Point-Of-Service (HMO-POS) Medical Savings Account (MSA)

What are common vulnerabilities among the immigrant population include?

Common vulnerabilities among the immigrant population include: ◦socioeconomic background ◦immigration status ◦limited English proficiency ◦federal, state, and local policies on access to publicly funded health care ◦residential location - limited resources Stigma and marginalization*

Which regulatory act regulates manufacture, distribution, and dispensing of drugs with a potential for abuse?

Comprehensive Drug Abuse Prevention and Control Act (Controlled Substances Act)

When was the first master degree PA graduates in york college?

First Masters degree class entered Fall 2016 (Graduated Jan 2019).

How were the first PAs received?

First PAs were received by the public - useful position for veterans from the Vietnam War - Competence tested on the battlefield - Helped reduce physician shortage - using corpsmen prevented the transfer of workers from other healthcare careers (capitalized on the training of U.S. gov't)

What was Specialty Training?

First developed at the University of Alabama - 2-year program - Clinical year focused exclusively on surgery and surgical subspecialties - other programs offered specialized training in urology, orthopedics, and pathology in the U.S. - Early specialization programs phased off as the need for broad-based education was emphasized

What is the first responsibility of PA education? A. Responsive to the needs of society. B. Responsive to the needs of the Facility. C. Responsive to the government demand D. Responsive to the economical need

First responsibility: be responsive to the needs of society. PA education was Founded by and resides within Universities, which dedicate themselves to society, not a particular profession [facility].

Who are students in maturation period?

Fluctuation in the number of students applying to PA school The majority of students had significant prior clinical experience at the time* but later, people entering PA as their first career [those with no prior called health professional experiences]. Motivation focused on career, income, status and security

Subspecialty groups provide

Focused CME, networking opportunities, advocacy to support the profession a chance to give back through volunteering

What hospital may look for?

For example. 32-36

What are AAPA's strategic commitments [3]?

Foster PA excellence [through integrating professional development and coordinated advocacy] optimize PA practice [Identify, evaluate and promote best practice models that increase PA impact on health, maximize PA value and enhance PA satisfaction.] grow the PA profession [elevate PA role, recognization, respect, and support the growth]

Key feature of PA education Mid 1960s through 1970s What is devotion to Underserved Population?

Founding philosophy for many programs Some programs only focused on students who will work in inner-city, rural, poor, or American Indian populations }Programs made the efforts to recruit from underserving population A number of programs chose clinical sites that provide services to the underserved

Where does •Supplementary Medical Insurance Trust Fund from and what is it for?

From Congress, premium from Part B and D, and investment. Holds funds for Medicare Part B and Part D Pays administration fees Funds programs to combat fraud and abuse

Preventative Medicine Medicare Codes Welcome to Medicare IPPE ~ < 12 months of medicare enrollment First annual wellness visit ~ > 12 months of medicare enrollment

G0439 is the most common code. Know that MEDICARE has their own code

When was the last bachelor degree PA graduates in york college?

Graduated in 2017 and admitted in 2015.

What are your chance getting HIV, hep B, hep C?

HIV - 1/5 Hep B - 1/50 Hep C - 1/300

Who must do the HPI?

HPI needs to be done by the clinician [has to have input in HPI] to ensure ... intuitive questions, pertinent detail.

What is the most important thing you can do to prevent infections?

Hand hygiene

What did Dr. Stead propose when the Advance Nursing Program was rejected?

He proposed to the Duke's Hopsital Administrator his intention of developing the PA program, using former military corpsman, modelng the relationship between Dr. Johnson and Buddy Treadwell

What is health disparities?

Health disparities refer to differences in the health status of different groups of people.**

What were tried to improve health disparity?

Health promotion/Disease prevention Access to low cost, high quality health care

What is the default assumption of patient capacity?

Healthcare workers will assume patients have decision-making capacity by default [Decision made base on conversation and assessment]

The Name Controversy for PAs

Henry Silver: wanted Syniatrist, as it means (syn= with, in association; iatrist= relating to medicine) but might be confused with a psychiatrist Richard Smith: wanted to avoid assistant or associate; wanted Medex

What are sharps injuries are primarily associated with?

Hep B virus (HBV), Hep C virus (HCV), and human immunodeficiency virus (HIV)

What are Schedule II-potential for severe psychic or physical dependence?

High abuse, efficacious Tightly controlled prescribing (no refills) Morphine, Methamphetamine, Hydromorphone (Dilaudid)

Where does hospital Insurance Trust Fund from and what is it for?

Hospital Insurance Trust Fund ◦Hold funds for Medicare Part A ◦Funds the administration costs of Medicare ◦Funds Medicare tax collection ◦Funds programs to fight Medicare fraud and abuse }Hospital Insurance Trust Funded by ◦Payroll taxes paid by employees, employers and people who are self-employed ◦Income taxes on Social Security Benefits ◦Interest earned on trust fund investments ◦Medicare Part A premiums - for those who are not eligible for free Medicare Part A

What is the definition of house officers?

House officers ~ putting out fires ~ pt. has pain, go evaluate, get paged, go to another. Usually at night. GOOD EXPERIENCES ~

Where to look up ICD 10?

ICD 10 Look-up }CMS ICD 10 Resources https://www.cms.gov/Medicare/Coding/ICD10/ICD-10Resources }ICD 10 Look up Tool }http://www.icd10data.com/

Preparing Collaborative Practice Ready Health Care Professionals...

IPE -> collaboration -> improve patient care coordination -> Better patient outcome!~

Questions PAs should ask Are they familiar with the legalities of working with a PA in their state?

If new to PAs, are they committed to setting up your team practice properly in accord with regulatory agency requirements?

Where can a patient file complaint when privacy vilolated?

If patients feel their privacy has been violated, they can file a complaint with: ◦Provider/Organization > ◦Health plan -> ◦Department of Health and Human Services (HHS) -> ◦Office of Civil Rights

Who is considered as a vulnerable population?

Immigrants - a group at increased risk for poor physical, psychological, and social health outcomes and inadequate health care.

What program did Eugene Stead, MD develop?

In 1950s-1960s, Eugene Stead, MD developed a program to extend the abilities of RNs at Duke University - this was opposed by National league of Nursing (didn't want to transition nurses to medical model) **At this same time, Duke University began training several firemen, ex-corpsmen and non-college graduates to help solve clinical service shortage (Duke PA program arose in a time of healthcare crisis)

What did Henry Silver, MD and Loretta Ford, RN create?

In 1965, they created a practitioner training program for nurses for impoverished pediatrics - beginnings of NP movement

What names did the AMA propose for PAs?

In 1970, AMA turned to congress to end controversy Congress initially proposed "associate" - indicated a more collegial relationship and eliminated confusion with medical assistant AMA opposed the "Associate" title because it was too close to another doctor and stated "associate" should be reserved for physician/physician relationship Eventually chose "assistant"

When was the name "Physician Assistant" confirmed?

In 1980s - the AAPA recommended that graduates of AMA accredited programs refer to themselves as physician assistants

AAPA Annual Salary Report 2020

In 2019 compensation increased in the PA profession by 3.3%.

What can electronic prescribing do?

In NYS, E-prescribing can be used for both controlled and non-controlled substances Mandatory Electronic Prescribing Initial date On March 13, 2013 Effective - March 27, 2016 Clinicians must register for an electronic prescription for controlled substances (EPCS)

What tests should you consider testing patient when pt present with fever, cough, sore throat (new loss of taste or smell)?

CoVID 19 and influenza

What concept does the PA profession found on?

Collaborative practice but not independent practice.

What are two main types of codes in ICD?

Diagnosis codes Procedural codes

Why was there decrease in program in period of pioneer period?

Lost of funding, issue with accreditation, external forces.

What are diseases that require airborne precaution?

M. TB Chicken pox Herpes zoster

What is MCO - Managed Care Organization?

MCO - Managed Care Organization ◦An organization that delivers managed care PPO HMO Fee-for-service programs

Which organism should you consider the use of contact precaution?

MRSA, VRE, C. Diff, Scabies, generalized rash

When did electronic prescribing become mandated?

March 2013 EFFECTIVE: March 2016

What is American Health Benefit Exchange?

Marketplace for any American to purchase health insurance Forum for small businesses to purchase insurance for employees Premium subsidies for low-income families

Vicodin, Codeine, Anabolic Steroids are what schedule of drugs?

Schedule III Lower abuse, efficacious Contains less than 90 mg of codeine per dose unit

What did prior training or military corpsmen include?

- up to 1900 hours of formal medical training - 9 weeks of supervised clerkships - 3 to 20 years of active duty experience (aboard ships or on rural outposts) These military corpsmen were unable to work medically after discharge!

What models are the PA profession based on?

1. Feldshers in Russia 2. Barefoot doctors in China Many origins for the PA profession, although it is thought to be an American Concept

What are the goals of ARC-PA

- foster excellence in PA education - foster excellence in PA programs - assure public that PAs are ready and prepared for practice - provide information and guidance to individuals, groups, and organs regarding PA program education

What are two origin models for the creation of the profession?

1. Feldshers in russia 2. barefoot doctors in China

Why did PA programs expand again in 1990?

- issues of efficiency of medical education - need for team practice - cost-effective solutions to healthcare - Need to increase the availability of healthcare

Why is medical errors under recognized cause of death?

- no ICD code assigned for it - unintentionally excluded from the national health statistics

AAPA description of the profession:

"Physician Assistants are health professionals licensed or, in the case of those employed by the federal government, credentialed, to practice medicine with physician supervision."

What are social determinants?

"Social determinants of health are conditions in the environments in which people interact with on a daily basis that affect a wide range of health, functioning, and quality-of-life outcomes and risks." }It is important to recognize the impact that social determinants have on health outcomes of specific populations. ◦Conditions Refers to social, economic, and physical factors (conditions in which people are born, grow, live, work and age) ◦Environment Refers to place - eg. school, church, workplace, and neighborhood

What is PAEA mission?

- pursue excellence - foster faculty development - advance knowledge in regards to patient-centered care and quality education - promote diversity in all aspects of PA education

What has AAPA worked on in the past regarding to supervision requirements? [2]

# of physicians to PA ratio # of hours required for physician to be on site or close proximity.

Which one is the newest ICD?

(ICD-10) - International Classification of Diseases 10 }ICD - 10- CM ◦CM = Clinical Modification ◦Developed by the World Health Organization (WHO) }ICD 10 replaces ICD 9 on October 1, 2015 ◦The higher level of specificity }ICD 10 affects diagnosis and inpatient procedure coding for everyone ◦Does not affect CPT coding for outpatient procedures

ICD 9 VS ICD 10

(International Classifications of Diseases) ICD-9 has 3-5 alphanumeric for inpatient and outpatients with 3-4 numeric procedure codes; ICD-10 has 3-7 alphanumeric for inpatient & outpatients with 7 alphanumeric procedure codes

What are informed consent for treatment?

* All adult patients have the right to make treatment decisions as long as they have the capacity - which is: ◦The ability to understand and appreciate the nature and consequences of health care ◦The ability to reach an informed decision ---> Gives hospital blanket consent for treatment upon admission. *All specialized procedures require informed/written consent

Preventative Medicine Codes

***Depend on age and patient first time or follow up, codes will be different 993__8__1 -> 993__8__7 - new patient 993__9__1 -> 993__9__7 - established patient

How many types of history are under The levels of E/M services?

***}The levels of E/M services are based on four types of history A. Problem-focused PF B. Expanded problem focused EPF C. Detailed D. Comprehensive

What is the national PA day and week?

**National PA day October 6 First celebrated 10/6/1987 (at the 20th anniversary)

What is In NYS - Prescriptive privileges Inpatient settings?

*PA may write for any medical order *Orders may include controlled substances

How does PA profession was set up to benefit for these business populations?

*PA profession was set up to benefit from some aspect of this environment ◦Cost-effective [cheap] ◦High-quality medical care providers ◦Team practice ◦Patient-centered focus

What are role of PAs in Home Visits?

*PA usually employed by private practice to do home visit[before] but now more hospital and LTC uses PAs to do home visits. - Visit patient in their homes }Complete H&P, assessment and plan, order meds, referrals (including sending patient to hospital in case of acute conditions), draw blood, order diagnostic tests }Follow-up on all testing and with patient }Coordinate care with you supervising physician

What are the key aspect of social influences in The 1990s - Boom or Bust Time? A - movement of healthcare toward larger delivery system B - rising healthcare cost and need of affordable healthcare C - patients' need to travel farther for care D - Diminished the service-oriented values for patient care E - All of above

*Steadily rising healthcare cost and need for corporations to find affordable healthcare for their employees - Movement of healthcare toward larger delivery system - Small rural hospitals closed forcing patients to travel farther for care - Many small physician offices ◦Were managed by larger hospitals or larger group practices ◦Diminished the service-oriented values for patient care - medically underserved population rapidly expanded GoAL: Decrease ratio of specialty care to primary care. Incentive from government funding. **Create a majority of primary care provider due to rapid expansion of medically underserved population.

What are Purposes of HIPAA? }Provide protections for personal health information }Allow patients access to their health information }Increase trust in health care system }Protect patient autonomy } A - C

- A - C - Provide protections for personal health information - Allow patients access to their health information - Increase trust in the health care system

All of the following is true except? - Agent's decision is final - Agent is not financially responsible for the cost of your care. - Agent is not required to make health care decisions for you according to your wishes, religious and moral beliefs, and in your best interest. - Agent can decide how patients wishes apply as medical condition changes - Healthcare providers must follow the agent's decisions

- Agent is required to make health care decisions for you according to your wishes, religious and moral beliefs, and in your best interest. Agents decisions can be limited - if specified in the health care proxy document, e.g. ◦Antibiotics ◦Mechanical ventilation Artificial nutrition

Obj. 3 What is PA Program Accreditation Process?

- Apply for Accreditation - Provide evidence of an ongoing critical self-study process - Undergo and on-site evaluation - ARC-PA meets three (3) times annually to consider accreditation actions =March, June, September - Once accreditation is granted =Periodic reviews and on-site evaluations

When should you use S & W rather than alcohol based sanitizer?

- B & A using bathroom - Caring for people with known MRSA, VRE, C-diff - hand visibly soiled

What is blanket consent?

- Blanket consent for treatment signed on admission to the hospital. Here is a consent you sign and we can do the certain procedures without further consent EXCEPT for specialized procedure. *All specialized procedures require informed/written consent

What are the diseases that require droplet precaution? (6)

- COVID - FLU - Neisseria meningitidis - H. Flu meningitidis - Pertussis - Other respiratory virus (eg. Adenovirus, RSV, parainfluenza)

What immunization do PA students need for clinical rotations?

- CoVID19 - Hep B - MMR - Influenza

Who are qualify for educator since 2010?

- Educators are highly qualified - they hold masters and doctoral degrees - Improved technological skills

Where are PAs in Hospital In-Patient Settings?

- Employed in all areas of the hospital ◦Medical, surgical & intensive care units ◦Emergency room, Fast-Track ◦Postanesthesia care unit (PACU) ◦Radiology (Interventional) ◦Employee health

what are some key points that you should be aware in clinical setting as PA-S?

- Immunization - Be safe - Be aware of limitation - Be familiar with facility policies and emergency contact information

How to protect PA-students in clinical setting?

- Immunization - TB testing [PPD/Quantiferon] - Information on safety policies

Why did PA program expansion began again in 1990?

- Issues of efficiency of medical education - Need for team practice - Cost-effective solutions to healthcare - Need to increase availability of healthcare

From 1965-1985, why then 25 PA programs subsequently close?

- Lack of funding by institution - Withdrawal of accreditation - Adverse pressure from other health care groups

When did medicare began reimburse PA for their services?

- Medicare began reimbursing PAs for their services in 1977 - ◦authorized by the Rural Health Clinic Services Act - Later expanded Medicare Part B payment for services provided by PAs, coverage in: Hospitals Nursing facilities Rural Health Professional Shortage Areas and First assisting at surgery. }In 1997, the Balanced Budget Act extended coverage to all practice settings at one uniform rate.

What are things needed for airborne precaution (pt, you, and environment)?

- N95 mask - negative pressure room - Pt wear a surgical mask during transport

What two laws are statue for New York Pas?

- New York Education Law - New York Public Health Law

What is necessary for EBOlA universal precaution? [in terms of equipment and body coverage and training]

- PPE must covers entire body - comprehensive training on Ebola-related infection control practices & procedures. - Supervised by onsite Manager

Key feature of PA educationMid 1960s through 1970s What is Psychosocial Emphasis?

- Physicians lack the interpersonal skills at the time, PA can spend more time with Pt. - PA trained to have patient-centered focus, interviewing, and counseling skills.

Which duties cannot be delegated to PAs in NYS?

- Taking radiographs (X-rays) - Performing ocular refractions unless currently licensed as an optometrist, optician

What are considered contact precaution procedures?

- Wear gown, glove - Wash hands with soap and water - Sanitize equipment used.

What might need to provide to the patient when conducting disclosure of medical errors? (5)

- emotional support and acceptance - truthful information in a timely fashion - to understand that multiple discussions may be required to address all questions. - medical information in appropriate, but jargon free language. - to understand what happens next.

What does leadership style reflect on?

- the nature of that organization - its relationship with the community.

How to maintain Confidentiality- HIPAA?

-**Patients have the right to privacy and confidentiality. -**Always use a private place for: ◦Case discussion and consultation ◦Patient examinations and treatment -**A patient's medical record may be shared with: ◦Clinicians directly involved in the patient's case. [nurse, consultant]. ◦Regulatory agencies looking into a facility's quality of care [insurance company] ◦Other people with legal or regulatory right to see the record. [i.e. legal authority] -**Only authorized employees should have access to where medical records are stored.

leadership style: using relationships

-DEVELOP STRONG RELATIONSHIPS with others and use this relation to steer people in a particular direction -people do the work b/c of their relationship with their leader

What are Maturation Period Social Forces influencing this period the 1980s A Development of different groups B Healthcare managed as a business C Physician specialists discovered the benefit of PAs D Hospital facing issues such as a reduction in residents and fluctuation of quality of care. E All of above.

-Development of: ◦Health maintenance organizations (HMO) ◦Preferred provider organization (PPO) ◦Diagnosis-related groups (DRGs) Healthcare was to be "managed" as a business (for-profit) **PA profession was set up to benefit **Developed new systems - Focusing on geriatrics, medical home visits , patient education, case management and comprehensive occupational medicine. **Physician specialists also discovered PAs **Hospitals were struggling and faced with many problems ◦Reduction in residents ◦Fluctuation in quality of care as residents rotate ◦Need for support to attending physicians ◦Hospital discovered and utilize PAs

What is staff model of HMOs?

-HMO employs its own providers, paying a fixed yearly salary [HMO own facility, e.g. HIP owns HIP center and sees pt in HIP facility] ◦Physicians only see patients that signed up for the HMO that employs them. - has their own building for office

Other Models for PA development include:

-MEDEX model ~ Richard Smith, MD. -Child Health Associate Program Henry Silver MD -Specialty Training

How to develop your OWN leadership style?

-Start with yourself -Think about the needs of the organization or initiative -Be prepared to change -Believe in what you are doing -Observe and learn from others -Use the research on leadership

What are some characteristic of servant role? (10)

-a few characteristics = listening empathy healing awareness persuasion conceptualization foresight stewardship toward need of others commitment growing of people building community

When are challenges more prominent?

-challenges are more obvious when: something new is about to start something is about to end tough times during transition

When is default assumption of patient capacity ruled out?

-default assumption is patient has capacity except when a patient has appointed a legal guardian [when pt has limited capacity]

What does servant leadership emphasize?

-emphasizes: >>a holistic approach to work >>promoting a sense of community >>sharing of power in decision making >>increased service to others

T/F, a good leader always sticks to one specific style and does not change.

-good leaders have a specific style but are flexible -be consistent with what people in the organization expect

All are external challenges to leadership except? -public criticism -interpersonal issues within the team -crises -opposition/hostility from powerful forces -financial/political windfall -collaboration -impatience with others and situations

-impatience with others and situations a few examples: -public criticism -interpersonal issues within team -crises -opposition/hostility from powerful forces -financial/political windfall -collaboration

All are internal challenges to leadership except? -insecurity -defensiveness -lack of decisiveness -opposition/hostility from powerful forces -inability to be direct when there's a problem -inability to be objective -impatience with others and situations

-opposition/hostility from powerful forces a few examples: -insecurity -defensiveness -lack of decisiveness -inability to be direct when there's a problem -inability to be objective -impatience with others and situations

How to manage people? Again, manager main function is to produce order and consistently though process such as planning, budgeting, organizing, staffing, and problem solving.

-planning: setting clear goals, understand the skill level for each employee -delegating: share goals with the employees (big picture), assign tasks based on employee's talents, true delegation puts the employee in charge -motivating: find out what motivates/drives employees

How to determine appropriate style of leadership?

-style needs to be consistent with the goals, mission and philosophy of your organization

leadership style: power and influence

-the LEADER IS CUNNING AND DECEPTIVE when necessary and the end justifies the means >>lead through manipulation >>creates friction (allies and enemies)

leadership: definitions

-the process of influencing an organized group toward accomplishing its goals - a process by which a person influences others to accomplish an objective and directs the organization in a way that makes it more cohesive and coherent -a process of social influence which maximizes the efforts of others towards the achievement of a goal

What are the two universal requirements for state Credentials (Licensure)?

1. Graduation from an accredited PA Program 2. Pass the PANCE (administered by NCCPA)

What are Functions of AAPA? [4] 2 to PA profession, 1 to PA-S, 1 to patient [1,4/2/3]

1. Advocates and Educates on behalf of the profession [provides CMEs and publish journal JAAPA] 2. ensure professional growth/personal excellence/recognition of PA profession 3. enhances PA ability to improve quality, accessibility, and cost-effectiveness of patient-centered care 4. Lobbies for PA legislation and also aid state chapters to improve local practice acts 2 to PA profession, 1 to PA-S, 1 to patient [1,4/2/3]

What are the roles/Fxns of ARC-PA?

1. establish educational standards 2. create a process for review of programs 3. Administer decision-making process 4. decide if programs are in compliant with standards 5. work with collaborating organizations 6. define a process for appeal of decisions made

What is a limited permit?

1. Did not yet take/pass PANCE 2. Expires in one year and can be renewed for another 3. Must be under DIRECT supervision of MD 4. $220 fee 5. Must be employed

How are the five drug schedules classified?

1. How dangerous they are 2. Potential for abuse and addiction 3. Whether they possess legitimate medicinal value

What concepts are the PA profession based on? [6]

1. Increased social consciousness (equality for all) 2. Greater demand for health services and rising health care costs 3. Physician shortage and their geographic and specialty maldistribution 4. Awareness of a variety of physician extender models 5. The availability of nurses and ex-corpsmen as potential sources of manpower 6. Increased need for clinical support professionals at local hospitals

Who can be evaluated by quality measure? (may be person or organization)

1. MCO 2. health plan or program 3. Hospital 4. Health care prtactitioner (Almost everyone)

What are 4 subsets of professional organizations?

1. National [ARC-PA, AAPA, PAEA, NCCPA] 2. State Chapter 3. Specialty 4. Interest group

What are limitation of PAs in US in ophthalmology?

1. Radiology - Xray taking, can not press the button unless has radiology tech certification 2. PAs are not authorized in NYC to write refractory or contact lenses.

What are three development in PA education in The 2000s - Moving forward?

1. Standardize professional degree, master should be awarded. 2. Development of Centralized application system for PAs as (CASPA) in 2001 3. International development of PA programs [PAEA]

What are the four core competencies of interprofessional collaboration?

1. VALUES/ETHICS for interprofessional practice 2. Roles/responsibility 3. Teamwork 4. Interprofessional communciation

}You are on your clinical Rotation }A patient was brought in by the police with lacerations to his hand, sustained during a fight }While suturing, the patient suddenly moved resulting in you (the PA student) sustaining a needlestick. What should you do?

1. Wash the wound with soap and water 2. Remember pt name 3. Ask pt regarding to sexual history and past history of HIV, Hep B, C, 4. obtain consent from patient for blood borne pathogen testing. 5. Inform supervising physician 6. Inform program with regard to incidence of needle stick injury.

What are the 3 factors that determine the PA scope of practice?

1. state law 2. PA education and experience 3. Facility policy and the needs of the patients

What are two things PA can not do?

1.Taking radiographs (X-rays) unless currently licensed as a radiologic technologist [CANNOT PRESS THAT BUTTON!!!] - one click, license gone - Only press that button when you are radiology tech certified ** 2.Performing ocular refractions unless currently licensed as an optometrist, optician, etc.

Which four areas of professional behaviors do PA's competencies for PA profession fits in?

1.◦Toward the patient, 2.◦Toward other professionals, 3.◦Toward the public 4. ◦Toward oneself

What models in the U.S. was the PA role based on?

1930s - Fed Prison System trained military corpsmen to extend services of prison physicians - US coast guard trained 800 purser mates (ship supervisors) to provide health care on merchant ships - 4 month program - program was eventually discontinued

What are considered the "developmental" years for the PA profession?

1957-1970

When was the CRA? And what did the Civil Rights Act do?

1964 - Removes socioeconomic barriers and discrimination against African Americans - Increasing the need for healthcare providers

When did the medicaid start?

1965

What year is Medicare established?

1965 }Federally administered and funded }}No restriction on providers, once they accept Medicare Provides healthcare for three groups ◦Elderly ◦People with end-stage renal disease ◦Some disabled persons

When does Accreditation dates back? When did the name from JRC-PA changed to ARC-PA?

1971 Undertaken by AMA's sub committee (committee on Education for Allied Health Professions and Services) Joint Review Committee for Educational Programs for the Assistant to Primary Care Physician (JRC-PA) - was formed 12/1971 The JRC-PA was renamed the Accreditation Review Commission on Education for the Physician Assistant (ARC-PA) - 1988

How have PA programs expanded?

1971-1973 - 31 new programs established - subsidized by the US gov't 1975 - 1282 graduates 1974-1985 - 9 new programs - 16,000 practicing PAs in the US by 1985 1965-1985 - 76 total accredited PA program sin the US - 25 closed due to lac of funding, withdrawl of accreditation, adverse pressure from other health care groups, etc. No money/accreditation/ext. forces

What years are considered as the "Establishing the Profession" year for PA profession?

1971-1980

When was PAEA developed?

1972

When were the first PA-C issued?

1975

When did the first PA class of York college admitted and when did they graduate?

2004 and graduated in 2006

When did infection control required for PA- students?

2008 }NY Legislation August 1992 requiring healthcare provider to receive infection control training every 4 years }Physician Assistants are among the many who require this training Since 2008 - PA Students also require infection control training

92% in 2020 PANCE pass rate, what happened?

24 passed, lesser

What are number of sub specialty orgs?

26 Support AAPAs knowledge base in specialty areas

As of December 2020, how many accredited PA programs are there?

268

How many years is the NYS License valid for ?

3 years

In NYS, how many credit hours and supervised clinical training is required before completion of a PA program?

32 credit hours 1600 supervised hours (40 weeks)

What is persaud's advice for PANCE?

4 - 6 weeks of intense study for PANCE If failed, self analysis, what is weakness, what need improve.

What are four time period for PA education development?

4 time periods: ◦Mid 1960s through 1970s - Pioneer Period ◦1980s - Maturation Period ◦1990s - Boom or Bust time ◦2000s - Moving forward - futures 2010 and up For each period, examine: ◦Social forces influencing the period ◦Key features of PA education ◦Characteristics of PA educators ◦Attributes of PA students

How many state chapter organizations exist?

57

What is the difference between hospice care versus palliative care?

6 months cut off - Most hospices limit admission to hospice for those with 6 months prognosis and the discontinuation of "curative" treatments. - Provides support and care for those in the last phases of life-limiting illness. - Recognizes dying as part of the normal process of living. - Affirms life and neither hastens nor postpones death. - Focuses on quality of life for individuals and their family caregivers.

What is the reimbursement for services rendered surgery by TRICare?

65

What is the reimbursement for services rendered by a physician in a similar location by TRICare?

85

What rate does PA get paid for reimbursement?

85% }As of January 1, 1998, Medicare pays the PA's employer for medical and surgical services provided by PAs in all settings at 85 percent of the physician's fee schedule. }Hospitals that employ PAs must bill for their clinical services under Medicare Part B. }There is no Medicare requirement for the physician to treat the patient or be physically on-site when the service is being billed to Medicare under the PA's (employer/practice) name.

Office Visits New Patients - E/M Codes

9920_

Established Office Patients - E/M Codes

9921_

46 yr male, new patient, come for routine physical, what code?

99386

7 yr male, new patient, come for routine physical, what code?

99393

what does leadership style encompass?

>>how they relate to others within and outside the organization >>how they view themselves and their positions >>to a large extent - whether or not they are successful as leaders

What is a fee schedule?

A list of specified fees a physician charges for services e.g. for H & P, insurance only pays up to $100. Even provider bills $300, only get 100. If has copay of 20 then insurance only pay $80. ◦A listing of the maximum fee that a health plan will pay for a service based on billing codes. This comprehensive listing of fee maximums is used to reimburse a physician and/or other providers on a fee-for-service basis

Align each leadership style to correct descriptor A. The Facilitator B. Power and Influence C. Follower-Turned-Leader D. The Great Man or Woman E. History's Slaves F. Using Relationships G.Passive Resistance/Setting the example 1. ◦The leader is cunning and deceptive when necessary, and the end justifies the means. (Machiavelli, early 1500s) 2. ◦The leader allows the group to run by itself and not intervening unless necessary. (Laotzu, circa 500 BC) 3. ◦The best leader rises from the experiences of being a follower. (Aristotle, circa 350 BC) 4. ◦With self-discipline, self-control, and self-purification, the leader is followed by example only, not as a person. (Gandhi, early 1900s) 5.◦One who is born to lead, due to innate qualities of intelligence and personality leading all to subordinate themselves to him or her. (Carlyle, early 1800s) 6. ◦Develop strong relationships with others and use this relation to steer people in a particular direction 7.◦The situation dictates that a person finds himself or herself leading out of necessity. (Tolstoy, late 1800s)

A2, B1, C3, D5, E7, F6, G4 }The Facilitator ◦The leader allows the group to run by itself and not intervening unless necessary. (Laotzu, circa 500 BC) }Follower-Turned-Leader ◦The best leader rises from the experiences of being a follower. (Aristotle, circa 350 BC) }Power and Influence ◦The leader is cunning and deceptive when necessary, and the end justifies the means. (Machiavelli, early 1500s) Lead through manipulation Creates friction (allies and enemies) }The Great Man or Woman ◦One who is born to lead, due to innate qualities of intelligence and personality leading all to subordinate themselves to him or her. (Carlyle, early 1800s) History's Slaves ◦The situation dictates that a person finds himself or herself leading out of necessity. (Tolstoy, late 1800s) Using Relationships ◦Develop strong relationships with others and use this relation to steer people in a particular direction ◦People do the work because of their relation with their leader }Passive Resistance/Setting the example ◦With self-discipline, self-control, and self-purification, the leader is followed by example only, not as a person. (Gandhi, early 1900s)

Some characteristics of Servant leader A}Listening B}Empathy C}Healing D}Awareness E}Persuasion F}Conceptualization G}Foresight H}Stewardship I}Commitment to the growth of people J}Building community 1Ability to foresee the likely outcome (Similar to conceptualization) 2Persuade rather than using authority 3sense of community for those working in a given institution 4Potential for healing one's self and others 5Understand and empathize with others. 6a commitment to serving the needs of others 7Deep commitment to listening intently to others 8General awareness, and self-awareness 9nurture the personal, professional, and spiritual growth of employees 10Looking at long term goals; beyond day-to-day realities

A7 B5 C4 D8 E2 F10 G1 H6 I9 J3 }Listening - Deep commitment to listening intently to others }Empathy - Understand and empathize with others. }Healing - Potential for healing one's self and others }Awareness - General awareness, and self-awareness }Persuasion - Persuade rather than using authority }Conceptualization - Looking at long term goals; beyond day-to-day realities }Foresight - Ability to foresee the likely outcome (Similar to conceptualization) }Stewardship - a commitment to serving the needs of others }Commitment to the growth of people - nurture the personal, professional, and spiritual growth of employees }Building community - a sense of community for those working in a given institution

Where can practice in other states be obtained?

AAPA ("State Laws for Physician Assistants") Constituent Chapters State Boards of Medicine State education and Health Departments

4 National PA Specific Organizations

AAPA (American Academy of Physician Assistants) ARC-PA (Accreditation Review Commission on Education for the PA) PAEA (PA Education association) NCCPA (National Commission on Certification of PAs)

What does AAPA, PAEA, ARC-PA and NCCPA represent?

AAPA ~ represents PAs ARC-PA ~ represents PA and the public to ensure the program meets educational standards for PA education NCCPA ~ represents the public by ensuring a level of quality and provides certification for PAs. PAEA - representing Physician Assistant educational programs, ensures and foster excellence in PA education

Who is ACA?

ACA - Affordable Care Act (ACA)

Which five general national organizations - not PA associated but supports PA profession with CME, networking, information and job resources?

AMA American College of Surgeons American Academy of Family Physicians American College Physicians, American College of Pediatrics

Who and what is accreditation body?

ARC - PA national organization -Current accrediting body for PA programs ◦Accreditation Is an external peer review process ◦Evaluates to determine if it meets the standards by the Accrediting body dates back to 1971, initially regulated by the Joint committee on education for allied health professionals and services in 1971 After was regulated by JRC-PA aka ARC-PA ~ Joint review committee for educational programs for the assistant to PCP. Renamed to ARC - PA in 1988

What does ARC-PA accreditation do to programs?

ARC-PA Accreditation Encourages programs to continually evaluate and improve ◦Process and outcome Help prospective students identify programs that meet nationally acceptable standards Protects programs from ◦Internal and external pressures to make changes that are not educationally sound **Involve faculty and staff in a comprehensive process of ◦Planning ◦Evaluation ◦Self-improvement

Which organization evaluate structural characteristics as part of their assessment?

Accreditation organizations (such as the National Committee for Quality Assurance or the Joint Commission on Accreditation of Healthcare Organizations) - evaluate structural characteristics as part of their assessment.

What are the standards used for?

Adhering to the Standards, PA programs ensure: ◦The commonality in the core professional curriculum ◦Students are equipped with the tools necessary for the practice ◦Allow programs to be creative and innovative in: Program design, curriculum delivery, and evaluation to achieve program goals and student learning outcomes

If provider is not known, the manager is inflating the bill, what type of penalty would you get?

Administrative Sanctioning ◦If fraud is the result of bad administration ◦Same penalties may apply!!!

What are the four types of advance directives?

Advanced directives are legal documents that outline a patients health care wishes if they are unable to make decisions on their own. In New York State four types of advance directives ◦Health Care Proxy ◦Living Will ◦Living will with Health Care Proxy ◦DNR

What is the definition of Adverse event?

Adverse event - an injury caused by medical management rather than by the underlying disease or condition of the patient

What is the definition of unavoidable adverse event?

Adverse events that are neither preventable nor ameliorable. Example - patient received appropriate Tx with Abx for first time - developed allergic reaction

What are the current (Since 2010) & The FutureSocial Forces influencing PA profession?

Affordable Care Act Technological advances ◦EMRs ◦Tele-health (increased use with COVID19 pandemic) Moving toward: [for holistic care of patients] ◦Preventative care ◦Patient-centered care ◦Home care visits ◦Patient-centered medical homes (https://pcmh.ahrq.gov/page/defining-pcmh) Focused on a specific model in delivering healthcare

All are Challenges arising from Leadership itself except? }Keeping an eye on and communicating, the vision. }Keeping every day under control while you continue to pursue the vision. }Setting an example. }Lack of decisiveness }Maintaining effectiveness over time. }Finding support.

Answer: }Lack of decisiveness a few examples: It is hard to ... }Keeping an eye on and communicating, the vision. }Keeping every day under control while you continue to pursue the vision. }Setting an example. }Maintaining effectiveness over time. }Finding support.

Why you have to fill out the Limited Permit and Registration Application at the same time of job application?

Application requires identification of physician [supervising]

What are ARC-PA benefit to students? [3]

Assure that program meets required national standards Recognize the students' education as sufficient Eligibility for professional certification, registration, and state licensure

What are the requirements for practice for NYS licensing for PAs?

At time of application: 1. at least 21 years of age 2. of good moral character 3. successfully completed a bachelors degree 4. completion of an approved program (32 hours of classroom work and 1600 hours of supervised clinical training) 5. Passed the NCCA examination (PANCE)

Approx _______ of Chinese medical Students were barefoot doctors A. 1/2 B. 1/3 C. 1/4 D. 3/4

B. 1/3

What year was the first Pa program started? A. 1961 B. 1965 C. 1964 D. 1970

B. 1965 October 1965 First Four students, all ex-Navy (Military Corpsmen) entered a 2-year program at Duke University

How long was the first PA program at Duke University? A. 2-3 months B. 2 years C. 1 year D. 4 months

B. 2 years

How long was the formal 19th century training for feldshers? A. 2-3 months B. 2 years C. 1 year D. 4 months

B. 2 years

Who created a practitioner training program for nurses for impoverished pediatrics (Bachelors Degree)? A. Dr. Richard Smith B. Dr. Henry Silver C. Dr. Charles Hudson D. Dr. Amos Johnson E. Dr. Eugene Stead

B. Dr. Henry Silver

Who created the first program to offer a post-graduate degree in PA Training and first child health associate program? A. Dr. Richard Smith B. Dr. Henry Silver C. Dr. Charles Hudson D. Dr. Amos Johnson E. Dr. Eugene Stead

B. Dr. Henry Silver

Who suggested the term Syniatrist? A. Dr. Richard Smith B. Dr. Henry Silver C. Dr. Charles Hudson D. Dr. Amos Johnson E. Dr. Eugene Stead

B. Dr. Henry Silver

}The ROS MUST be completed by the clinician ◦A. True? ◦B. False?

B. False

What does the following statement entail? - insurance pay per service provided A. Capitation B. Fee for service C. Explanation of benefit D. Network

B. Fee for service

What is the following statement describing? - provide reduced rate healthcare to an insurance company - Pt has more flexibility, without needing referral - Typically has a higher member premium. - Pays with bundled pricing rather than fee for service - Rarely use capitation A. HMOs - Health Maintenance Organization B. PPOs - Preferred Provider Organization C. EPOs - Exclusive Provider Organization D. POS plans - Point of Service E. CDHPs (Consumer Directed Health plan or HDHP (High Deductible Health Plan)

B. PPOs - Preferred Provider Organization ◦An association of healthcare providers and facilities who have agreed to provide reduced rate healthcare to an insurance company ◦ ◦Can be owned by hospitals, insurance companies, physicians, and HMOs! ◦ ◦The insurance company collects monthly premiums ◦ ◦PPO makes a profit by charging insurance companies access to the network Establish a network of contracts with providers and services in an area Establish specific pay rates [bundle pricing rather than fee for service] ◦Contracting physician usually cannot negotiate }Control cost with utilization management (appropriateness of services provided) } }Allows consumers to go out of network at lower cost (vs paying out of pocket). } }Usually have higher premiums Control cost with utilization management - appropriateness of services provided [would not pay if not appropriate] }Rarely use capitation as payment method

◦Patient rights upon discharge are true EXCEPT!!! A. All patients must receive a written discharge plan B. Still need discharge plan when signing out Against Medical Advice (AMA) C. Important Message from Medicare if medicare patient D. All of the above are correct

B. Signing out Against Medical Advice (AMA) will not receive discharge plan. ◦All patients must receive a formal discharge notice Except- Signing out Against Medical Advice (AMA) }Rights upon discharge ◦All patients must receive a written discharge plan ◦Medicare patients get a copy of "Important Message from Medicare" ◦All patients must receive a formal discharge notice

Other ways of looking at leadership style: Correlate leadership style to correct response: A. Transactional leadership: B. Charisma C. Transformational leadership D. Persuasion E. Sharing power. F. Involving followers in the goal. G.Situational or Contingency Leadership Models 1. Pull others along with the power of their personalities alone.. Able to advocate and accomplish wonderful things, but they do it through people's loyalty to and awe of them. 2. Followers are motivated in an exchange relationship by internal and external rewards to accomplish the goal AND views leadership based on transactions between leader and followers. 3. Someone who can distill the values and hopes and needs of followers into a vision. Others are empowered by the leader with a shared vision that meets the immediate needs of the group while incorporating larger, long-term goals that develop and grow the organization 4. Convinces people through argument, reasoning, selling techniques, or other persuasive methods that what the leader wants is, in fact, the best course, or in line with what they want to do. 5.The leader gets others to buy into her vision for the organization, and to make it their own. 6.Leadership is based on the position of power, the interactions between the leader and members, and the desired goal - Eg. Running a code 7. Some leaders choose to exercise at least some leadership through the other stakeholders in the organization.

B1, D4, A2, C3, E7, F5, G6

How to comply with HIPAA?

Basically don't do anything stupid }Share protected patient information and discuss cases only with people who are directly involved in the patient's care. }Do not gossip about patients. }Discuss cases in private. }Do not leave patient's charts out where they might be seen. }Do not display protected information where it might be seen.

What happened to the R in RPA-C?

Became redundant, meant registered but if you were certified you were registered Now instead of register with NYS, we get licenses from DOH.

When should you wash your hands?

Before entering and leaving patient's room Touching anything contaminated Before putting on and after removing gowns

What are bridge Programs for?

Bridge Programs ◦BS to MS bridge ◦Focus on Research

What is inter-professionalism and working collaboratively in interprofessional patient-centered teams mean?

Bring better health care and patient centered care

When did ARC-PA revised the standards on PA educational program and NCCPA updated and improved the certification of exams?

By 1990 the bust time.

When was PA accepeted by medical society?

By the end of 1980s the profession has grown sufficiently to become a part of the healthcare system. PA program becoming permanent unit lead to creation of tenured track position for faculty ◦Commitment to obtain graduate degrees ◦Faculty focus on educational/Academic career

Where is medicaid administered by?

By the state and is federally funded. No restriction on providers, once they accept Medicaid. Except for Medicaid HMO - must go to PCP.

All of the following is correct except _____. A. Entity must develop risk management measures B. Remote access risks do occur with access, storage, and transmission of information C. Training is optional to employees regarding passwords, protecting remote devices, the transmission of EPHI.

C!!! Not optional but must! }Training must be provided to employees regarding passwords, protecting remote devices, the transmission of EPHI. }Entities (organizations [private office, any facility]) must develop risk management measures, policies, and procedures to protect private information. Remote access risks occur with access, storage, and transmission of information

Who proposed the concept at an AMA conference - for the training of dependent "assistants to doctors"? A. Dr. Richard Smith B. Dr. Henry Silver C. Dr. Charles Hudson D. Dr. Amos Johnson E. Dr. Eugene Stead

C. Dr. Charles Hudson Although his proposal was denied, the concept of a Physician Assistant and Mid-Level Provider emerged

Where was the first specialty training program developed? A. Duke University B. University of Colorado C. University of Alabama

C. University of Alabama 2 year program Clinical year focused exclusively on surgery and surgical subspecialties

Scope of practice determined at the ____________ level A. National B. Federal C. Practice D. Physician

C. practice level

Correlate leadership style to correct response: A. leadership style: collaborative B. leadership style: autocratic C. leadership style: managerial D. leadership style: democratic 1. tries to involve everyone in the organization in leadership, initiates discussions, pinpoints problems, and keeps track of the organization as a whole 2. the leader who concerned primarily with the running of the organization 3. insist on doing it themselves, they have all of the power, make all of the decisions, and don't often tell anyone else about what they are doing 4. looks at his and others' positions in terms of responsibilities rather than status and often consults in decision-making

C2, D4, A1, B3

Which is always required for history component? CC HPI ROS PFSH

CC - chief complaints

What is CDHPs - aka High Deductible Health Plan (HDHP) plans?

CDHPs - aka High Deductible Health Plan (HDHP) plans ◦Higher deductible plans coupled with a Health savings account (HSA) ◦Chosen by individuals in good health - usually younger people ◦Healthcare cost paid from pre-tax savings account (Ex: HSA) ◦Additional costs are paid out-of-pocket until the deductible is met ◦Cost savings for both Employers and Employees

General independent national organizations help with what?

CME, networking, information and job resources

CPT or ICD? }Patient in ER with acute systolic CHF; Echo performed done showed LVEF of 30%. }Patient with fever and cough, found to have PNA with COPD exacerbation }Patient came to your office and had an incision and drainage done for abscess in right groin }Duplex US performed in your office }Patient with SOB, fever, chills, dysuria and chest pain found to have sepsis from PNA and UTI. Also with urinary retention. A foley was placed.

CPT or ICD? CPT and ICD - }Patient in ER with acute systolic CHF; Echo performed done showed LVEF of 30%. ICD - }Patient with fever and cough, found to have PNA with COPD exacerbation CPT and ICD - }Patient came to your office and had an incision and drainage done for an abscess in the right groin CPT - }Duplex US performed in your office CPT and ICD - }Patient with SOB, fever, chills, dysuria, and chest pain found to have sepsis from PNA and UTI. Also with urinary retention. A foley was placed.

What is role of PA in influences on Health disparities?

Can be a culturally sensitive health care providers to provide better health

What are the influencing factors of social and socio-economic affecting health outcome?

Can be either positive or negative }The influence of social and socio-economic factors on health/health outcomes include: ◦Demographics (age, gender) ◦Educational attainment [high ed. better outcome] ◦Income ◦Employment ◦Community "ICE-ED" }Protective Social factors include: ◦Social Support ◦Self Esteem ◦Self efficacy

Medicare ~

Care for older

Medicare and Medicaid What type of agency is centers for Medicare and Medicaid Services and what does it administers and monitors?

Centers for Medicare and Medicaid Services (CMS) Is a Federal agency ◦CMS is a branch of the Department of Health and Human Services (HHS) CMS administers the Medicare program CMS monitors Medicaid programs ◦Medicaid is offered/administered by each state

When did major change in the law effect PAs? what was it?

Chapter 48 of the Laws of 2012 Signed by the Governor on June 1, 2012; took effect January 1, 2013 Separates Physician Assistant and Special Assistant in the law PAs in New York are authorized to sign death certificates

Who was Charles Hudson and what did he propose?

Charles Hudson, MD (1961) ( a newly elected AMA trustee member) proposes the concept at an AMA conference- for the training of dependent "assistants to doctors" These assistants would be able to: - Lumbar puncture - Suturing - Intubation (This proposal was denied, but this concept of mid-level provider emerged)

What medicaid program is specific for NYS?

Child Health Plus - NY State [not just city] ◦Must be resident of NY State ◦Must be under the age of 19

What is CHIP stand for and who is it for?

Children's Health Insurance Program ◦Created in 1997, provides financial assistance for children of families that do not qualify for Medicaid

When were barefoot doctors seen?

China's Cultural Revolution in 1965 In response to Chairman Mao's directive to reorganize healthcare (Chairman of Communist Party of China) Trained 1.3 million "barefoot doctors" over 10 years

What is clinical post graduate programs?

Clinical post graduate programs ◦12-24 months ◦Medical or surgical specialties ◦Association of Post Graduate PA Programs - https://appap.org/ ◦ARC-PA Accredited Programs - revised 3rd Standards to be effective 1/2020

What is EPCS and how to verify controlled substances prescription?

Clinicians must register for an electronic prescription for controlled substances (EPCS) electronic prescription for controlled substances (EPCS) Using EPCS which gives Token or software NEVER SHARE EPCS TO OTHER PHYSICIANS! IF NOT YoUR PT AND YOU DON'T KNOW HIM, NOPE!

Which act establishes five schedules of controlled substances?

Comprehensive Drug Abuse Prevention and Control Act(Controlled Substances Act) CSA regulates the manufacture, distribution, and dispensing of drugs with a potential for abuse Established the five schedules that classify controlled substances as to: ◦How dangerous they are ◦Potential for abuse and addiction ◦Whether they possess legitimate medicinal value }Provides legal framework for the DEA

What are two types of social determinants?

Condition and environment

What is the issue with Consumer-Directed Health Plans?

Consumer-Directed Health Plans Increases consumerism in healthcare ~ Individuals make healthcare decisions based on cost and quality of services ◦Often integrated with PPO type models More flexibility for patients Ex: Blue Cross Blue Shield

}You are a PA practicing in emergency medicine - was driving home from work after an overnight shift, saw an accident on the road and stopped to help. ◦You did everything you could to help the patient including performing CPR - by the time EMS arrived, the patient had regained consciousness ◦Patient was taken to the hospital and survived, but with multiple disabilities ◦Few months later, the patient decides to sue you, since she had a DNR in place and didn't want to be resuscitated. ◦ ◦What should you do?

Contact facility for malpractice insurance and in NYS, is covered good Samaritan law - PROVIDING NOT GETTING ANY COMPENSATION!

What are types of precaution?

Contact precaution Airborne precaution Droplet precaution Universal precaution

What are some thing NPs and PAs share?

Contribute significantly to healthcare Believes their roles are cost-effective solutions to the healthcare crisis. Work well as a team

What should you not do when offering an apology?

Do not blame the system or the colleagues!!! Be careful with "BUTS"

What is the supervisory ratios for NYS prior to March 2013 for correctional facility? what about after March 2013?

Correctional facility prior to 2013: 4 PA to 1 physician after 2013: 6 PA to 1 physician

What is the MEDEX model?

Created by Dr. Richard Smith ~ 1965 - 1971 "Medical Extension" = Placed Clinical phase students in rural areas with the expectation of employment after training (due to shortage) - emphasized a "receptive framework" for the new profession and created relationships with legislators, regulators, and third-party payers **This was to facilitate acceptance and utilization of this new profession

What is the Child Health Associate Program? [CHAP]

Created by Henry Silver, MD - focused on pediatrics - offered medical education those with at least 2 year college degree and no medical experience - accepted non-military candidates - Bachelors at 2 years, and Masters at 3 years - First program to offer a post-graduate degree in PA training

What are medicaid created for?

Created to cover three groups ◦Low-income individuals - Healthy women and children makeup 70% of Medicaid covered population ◦Aged and younger persons with a chronic debilitating illness (Disability) ◦Institutionalized individuals - Nursing homes - Facilities for developmentally disabled and mentally ill

What is criteria for shared visit?

Criteria for shared visit ◦PA and Physician must be employed by the same employer (practice, group, hospital or corporate entity) ◦PA and Physician must see the patient in the same calendar day [Combined services into one claim that was delivered to the same patient] ◦The shared visit concept does not apply to procedures, critical care services, or nursing home visits. ◦Shared visits can be applied to initial and subsequent hospital visits, as well as visits in the Emergency Department [recall outpatient needs pt to be seen by the physician first]

What are other names for cultural competency?

Cultural awareness and cultural sensitivity and cultural humility

What is culture?

Culture is often described as the combination of a body of knowledge, a body of belief and a body of behavior.

Which state covers medical services provided by PAs under their Medicaid fee-for-service or Medicaid managed care programs?

Currently, all 50 states and the District of Columbia cover medical services provided by PAs under their Medicaid fee-for-service or Medicaid managed care programs. - The rate of reimbursement is either the same as or slightly lower than that paid to physicians.

Why did a small number of institutions offering a Masters's degree in the maturation period? All are correct except A- work was equivalent to masters. B- Teaching faculty required a master's degree C- work performed by PAs in the field was similar to that of those with a Master's degree D- All of the above

D- All of above

When was the name physician assistant chosen? And by whom? A. 1961 B. 1965 C. 1964 D. 1970

D. 1970 AMA

When was the comprehensive Health Manpower Act (Section of the public health act) authorized support for PA training passed? A. 1961 B. 1965 C. 1964 D. 1972

D. 1972 Education of PAs for ambulatory medical care Deployment of PAs to medically underserved communities Recruitment of minorities and women into health professions Recruitment of larger numbers of residents from medically underserved areas

How long was the federal prison system program ( that was discontinued) that trained military corpsmen to extend services of prison physicians? A. 2-3 months B. 2 years C. 1 year D. 4 months

D. 4 months

What is the Pioneer Period PA Education features the Mid 1960s through 1970s approach based on? All of the following except A. Compact Curriculum B. Diversity of education C. Psychosocial emphasis D. Curriculum development E. Devotion to underserve population

D. Curriculum development ◦*Approach was based on the philosophy of: 1. Educating people to meet a specific societal need 2. Increase the availability of quality healthcare 3. Values - strong service-oriented ◦Several early features remained with PA education Compact Curriculum Diversity of education Psychosocial emphasis Devotion to underserve population

A patient was seen at 8 am in the hospital by PA Persaud for CHF exacerbation. At 5 pm, Dr. Jones saw the same patient for the same complaint. All of the following are true, EXCEPT? }A. This visit can be billed as a shared visit }B. The bill should be submitted under Jones' name and NPI number }C. Dr. Jones will be reimbursed at 100% rate }D. Dr. Jones will be reimbursed at 85% because he only had a very brief encounter with the patient.

D. Dr. Jones will be reimbursed at 85% because he only had a very brief encounter with the patient. Criteria for shared visit ◦PA and Physician must be employed by the same employer (practice, group, hospital or corporate entity) ◦PA and Physician must see the patient in the same calendar day [Combined services into one claim that was delivered to the same patient] -> thus, will get reimbursement of 100%

All are the Pioneer Period Social Forces influencing this period the Mid 1960s through 1970s EXCEPT? A. Society's need for improvement in quality medical care B. Concept of the new provider C. Expansion of health care delivery D. Hospitals were struggling and faced with many problems E. Millions of people obtain healthcare

D. Hospitals were struggling and faced with many problems - Happens in the maturation period ◦*Society's need for improvement in quality medical care ◦ ◦*Concept of new provider: Dx, Tx & Prevent a wide array of diseases Practice after 2 years of education ◦*Expansion of health care delivery Medicare and Medicaid established Millions of people obtain healthcare Already deprived profession cannot meet the demands of new patients Most deprived areas with this expansion are rural and inner-city

What is the following statement describing? ◦No cap on how much providers can bill. ◦Coverage initially only for illness (Catastrophic) ◦Regular premium paid by the enrollee to Ins co. ◦Company-paid what they felt was a maximum appropriate charge, enrollee had to pay anything not covered by insurance. ◦No cap on how much providers can bill. A. HMOs - Health Maintenance Organization B. PPOs - Preferred Provider Organization C. EPOs - Exclusive Provider Organization D. Indemnity Insurance E. CDHPs (Consumer Directed Health plan or HDHP (High Deductible Health Plan)

D. Indemnity Insurance

What is the following statement describing? ◦Reimbursement through capitated payments ◦Members retain coverage outside network although with less coverage ◦Primary care physician acts as gatekeeper to other services A. HMOs - Health Maintenance Organization B. PPOs - Preferred Provider Organization C. EPOs - Exclusive Provider Organization D. POS plans - Point of Service E. CDHPs (Consumer Directed Health plan or HDHP (High Deductible Health Plan)

D. POS plans - Point of Service }Combine characteristics of PPO and HMO plans ◦Reimbursement through capitated payments ◦Primary care physician acts as gatekeeper to other services ◦Members retain coverage outside network although with less coverage 60% as opposed to 100% coverage May also be responsible for co-payments, coinsurance and an annual deductible e.g. Pt with GHI, can see Doctors that take GHI insurance group. - can have out of network but only cover specific percent. }Became unpopular ◦Individuals did not want to risk need of going out of network ◦Out of network costs could become substantial for rare diseases } }Led to the creation of Point-of-Service HMOs

Which of the following is not PHI? A. health care to an individual B. health status C. Payment information D. Recent travel history

D. Recent travel history

What is DEA for?

DEA Registration }Necessary to prescribe controlled substances (Schedules II-V)

What is Drug enforcement agency number?

DEA# is not required for Non-controlled substances! DEA number (DEA Registration Number) is a number assigned to a health care provider allowing them to posses or write outpatient prescriptions for controlled substances ◦Allows prescription of class II-V medications ◦Physician ◦Physician Assistant ◦Nurse Practitioner ◦Pharmacist ◦Dentist

When should the expansion stop?

Dep on future supply and demand. ▫E.g. Implementation of Obamacare has drastically increase need Based on employment trend and health care projections

What are two type of penalty for HIPPA violation? Who enforces it?

Department of Health and Human Services (HHS) administers and enforces standards civil money penalties criminal penalties No need to memorize - Civil Money Range from $100 to $50,000 or more per violation. - Criminal penalties for knowingly violating privacy rule range from: ◦Up to $50,000 to $250,000 and ◦One to ten years imprisonment

Which organization administer and enforces standards of HIPPA?

Department of health and human services

Does hospice care includes hydration through tube feed?

Depend on the patient proxy/surrogates will. E.g. expect prognosis of life is poor, can withheld antibiotic, just keep morphine for quality improvement.

Learning Objectives

Describe various PA practice settings: ◦Private Practice ◦In-Hospital Settings & Long-Term Care Settings ◦Home Visits and Outpatient Settings Contrast and compare various models of healthcare delivery systems Describe various forms of health care insurance including Medicaid and Medicare Describe the components of the public health care system

What are the questions PAs should ask to find the right doctor? Questions PAs should ask

Does the physician listen and communicate well? How much do they know about PAs? Do you get the sense that they will be accessible and available? Are there collegiality, compassion and service guiding principles? ◦Or does the bottom line seem to take precedence? What are their clinical expectations of you? Do they fit with what you'd like to do in this position? Will there be opportunities for your growth and development? Do they seem like they'd be good mentors if you are in a new practice area? If you are a seasoned practitioner, do you sense they will give you appropriate autonomy?

What is AHRQ definition for health quality?

Doing the right thing for the right patient, at the right time, in the right way to achieve the best possible results.

*Patient Have Rights to know all except ____ A Diagnosis B Prognosis C Treatment Options D Right to refuse treatment E Request a provider with specific demographics - eg. Age, sex, gender, religion

E - Request a provider with specific demographics - eg. Age, sex, gender, religion **meaning patient DO NOT have the right to select whom to treat them. }Patients cannot request a provider with specific demographics - eg. Age, sex, gender, religion ◦But they do have the right to refuse treatment. }To make informed decisions about treatment, patients must be given accurate information. Patient has right to ... }Participate in decisions about their care }Set the course of their treatment }Patients can see and obtain a copy of their medical records.

What is false of the following notes regarding notice of Privacy Practices? A. Must document privacy policies and inform patients of these policies. B. Patient must acknowledge receipt of these policies C. Must have in place a procedure for the release of information D. All of the above is false E. All of the above is true.

E. All of the above is true. *Notice of Privacy Practices Covered entities (Facilities/Organizations) ◦Must document privacy policies [HIPPA] and inform patients of these policies. ◦The patient must acknowledge receipt of these policies ◦Must have in place a procedure for the release of information

What is the following statement describing? Higher deductible plans coupled with a health saving account [pre-tax savings account] ◦Chosen by individuals in good health ◦Cost savings for both Employers and Employees ◦Additional costs are paid out-of-pocket until the deductible is met ◦Chosen by individuals in good health - usually younger people A. HMOs - Health Maintenance Organization B. PPOs - Preferred Provider Organization C. EPOs - Exclusive Provider Organization D. POS plans - Point of Service E. CDHPs (Consumer Directed Health plan or HDHP (High Deductible Health Plan)

E. CDHPs (Consumer Directed Health plan or HDHP (High Deductible Health Plan)

Who proposed to Duke's Hospital Administrator his intention of developing the PA program, using former military corpsman, modeling the relationship between Dr. Johnson and Buddy Treadwell? A. Dr. Richard Smith B. Dr. Henry Silver C. Dr. Charles Hudson D. Dr. Amos Johnson E. Dr. Eugene Stead

E. Dr. Eugene Stead

What are different categories of E/M? [7]

EVALUATION AND MANAGEMENT CODES }E/M codes are divided into multiple categories: ◦Office or other outpatient services ◦Hospital observation ◦Hospital inpatient services ◦Consultations ◦Emergency department services ◦Critical care ◦Nursing facilities − initial and subsequent; discharge; rest home; home services; prolonged services; case management; care plan oversight; preventive and special evaluation and management.

What are the plans PAEA does to accomplish its vision [to improve the quality of health care for all people by fostering excellence in PA education]?

Educating competent and compassionate PAs Help to recruit well-qualified students and faculty - Facilitate research, professionalism, and innovation in health professions education - promotes inter-professional education - link with other organizations - Educate PAs who will practice evidence-based, patient-centered medicine. - Serve as the definitive voice on matters related to entry-level PA education, nationally and internationally.

What must the federally funded health facility must provide for all patient who request it?

Effective Communication }Effective health communication is as important to health care as clinical skill.

What does team approach support?

Efficient patient-centered healthcare.

What are three components of HIPPA title 2?

Electronic exchanges Security Privacy

What are Hospitals not required to do?

Employ PAs Grant privileges to PAs Change their bylaws to permit PAs to write medical orders

What year did New York Legislation begin for PAs?

Enacted 1972 (NYS began registering PAs)

What are the key aspect of PA education in Maturation Period? Postgraduate development

Entry-level programs 1. Limited to primary care 2. Exception of three general surgery programs 3. All other specialty were closed - Ortho, Urology, Radiology, etc. Development of Postgraduate programs in variety of specialty Ex - Occupational medicine, ER, General surgery, neonatology and geriatrics 1. Most were 1 year in length and focused on residencies 2. Most offer certificate of completion 3. Some offer a Master's degree

What is the following example considered as? Patient admitted with chest pain, EKG showed STEMI; patient not taken to cath lab immediately - eventually dies (Answer can be more than 1) Unavoidable adverse event? Error of omission? Negligence? Ameliorable adverse event? Preventable Adverse events?

Error of omission Negligence

What is the definition of error of omission?

Error of omission - occurs when a necessary procedure or intervention failed to be performed - leading to morbidity or mortality to the patient involved. Ex: Pt with fever, cough - failed to order X-Ray; patient later found to have severe sepsis from PNA, now intubated due to respiratory distress.

How many schedules of controlled substances are there? How were they established?

Established by Comprehensive Drug Abuse Prevention and Control Act (Controlled Substances Act) five schedules

Something about medicare

Everyone eligible for part A when older than 65. If has Medicaid for low income, within requirement, government pays for premium of part B. If not qualify Medicaid, you would have to pay premium of part B.

How did Feldshers provide and improve health care?

Feldshers provided healthcare: - on the battlefield After they complete their services: - remote Russian villages - Remote parts of Alaska **They continued practicing even after their retirement

Office Visits New Patients - E/M Codes 99203

Example: 99203 Detailed history Detailed examination Medical decision making of low complexity New pt. 63-year old female with H/O HTN presents for a pre-employment PE. BP has been adequately controlled with her current medication and home blood pressure monitoring. Requires: Review of Hx form completed by pt, and VS done by staff Obtain detailed Hx and detailed PE Consider relevant data, options, risks Formulate Tx plan (low complexity and decision making) Discuss Dx and Tx options Write Rx, order and arrange any necessary tests/referrals

Established Office Patients - E/M Codes 99212

Example: 99212 Expanded problem-focused Hx Expanded problem-focused PE Medical decision making of low complexity Office visit, established pt. 55 y/o male with H/O HTN and hyperlipidemia who presents for follow-up Requires: Review of Hx (form completed by pt), and VS done by staff Obtain expanded prob.-focused Hx (including response to Tx since last visit) and review of medical record. Expanded Prob-Focused PE Consider relevant data, options, risks and formulate Tx plan (low complexity and decision making) Discuss Dx and Tx options Address preventive care needs Write Rx, order and arrange any necessary tests/referrals

Example: 99212

Example: 99212 Expanded problem-focused Hx Expanded problem-focused PE Medical decision making of low complexity Office visit, established pt. 55 y/o male with H/O HTN and hyperlipidemia who presents for follow-up Requires: Review of Hx (form completed by pt), and VS done by staff Obtain expanded prob.-focused Hx (including response to Tx since last visit) and review of medical record. Expanded Prob-Focused PE Consider relevant data, options, risks and formulate Tx plan (low complexity and decision making) Discuss Dx and Tx options Address preventive care needs Write Rx, order and arrange any necessary tests/referrals

PAs as leaders in health care (4)

Examples -guiding pts to best health care decisions -managing a team of health care providers -leading a health care committee -overseeing community health programs

What are example of Medicaid Fraud?

Examples of Medicaid Fraud Include: ◦Billing for services not provided ◦Double Billing ◦Billing for Phantom Visits ◦Billing for more hours than there are in a day ◦Falsifying Credentials ◦Billing for unnecessary services or tests ◦Billing for more expensive procedures than were performed ◦Kickbacks ◦False Cost reports Penalties ◦Varies - dependent on circumstances Can be a criminal offense Jail time Fines Jail, penalty, revoke of certification

What are example of performance improvement?

Examples of Performance Improvement }Preventing re-admission for Heart failure, COPD, coronary artery bypass graft (CABG) }Medication reconciliation }Early identification and treatment of sepsis }Prevention (and reducing): ◦Catheter Associated UTI (CAUTI) ◦Central Line Associated Bloodstream Infections (CLAB) ◦Healthcare associated infections due to Multi-Drug Resistant Organisms (MDRO) }Glycemic control

No restriction on providers, once they accept Medicaid except?

Except for Medicaid HMO, which requires pt to must go to PCP.

When is the 21st Century Cures Act limited? A> Psychotherapy notes B> Consultation notes C> Procedure notes D> notes may be used in civil or criminal court case or administrative proceeding. E. B & D F A & D G all of above

Exceptions, include 1. ◦Psychotherapy notes 2. ◦Notes the provider has reasonable assumption could be used in a civil or criminal court case or administrative proceeding.

What are the key aspect of Eudcation in The 1990s - Boom or Bust Time? A - Increase in minority students B - expansion in all allied health and NPs healthcare profession C - Govn. provide funds. to PAs devoting to work in medically underserved area D - educator creating PACKRAT E - emphasis on Expansion, proliferation, and growth F. All of above

Expansion, proliferation and growth *More programs *More seatings

Question to ask What does level 1 and level 2 mean in E/M coding?

Explained in 45 99211 ~ Level 1 ~ simple 99212 ~ level 2 ~ UTI pt

What is the 21st Century Cures Act?

Extend notes information more than just written discharge plan. A portion of law that limits any kind of information blocking including blocking of patient data access. A portion of the law focuses on preventing any kind of information blocking - including blocking of patient data access. ◦Effective Nov 2, 2020 ◦Extended to April 5, 2021 The following 8 types of notes must be available to patients free of charge ◦Consultation notes ◦Discharge summary notes ◦History and physical ◦Imaging narratives ◦Laboratory report narratives ◦Pathology report narratives ◦Procedure notes ◦Progress notes

T/F PAs do not have the responsibility to actively seek out advice and consultation from a physician when needed.

F, It is the responsibility of the PA to seek advice and consultation when indicated. Although the physician is ultimately responsible for the acts of the PA, the responsibility to ensure that PAs practice in accordance with ethical, legal, and medical standards is shared and reciprocal.

Which of the following is not a means of supervision? A. Chart review and audit B. On-site consultation C. Periodic telephone contact D. Telemetry E. Shared practice days F. All of the above can be use to achieve supervision

F. All of the above can be use to achieve supervision

What are Benefits generally in hospital?

FYI }Benefits usually include: ◦Medical benefits ◦Dental ◦Vision ◦Tax Deferred Retirement options - some companies will match (up to about 6%) ◦Pension plan - provided by the facility at no cost to you ◦Vacation time - about 4 weeks ◦Sick time - about 10-12 days ◦Personal time - about 5 days ◦Tuition reimbursement - ranges from $4-12,000 per year ◦CME days off and reimbursement ◦Reimbursement for Certificates and Licenses ◦Maternity leave (3 months - job is guaranteed) ◦Bereavement leave - about few days

T/F, the document required for health care proxy needs two witnesses' signatures with a date. Both agent or alternative agent can be witnesses

False ◦Two witnesses signatures with date Neither agent or alternative agent can be witnesses

T/F, exposure is considered when you have sharp injury contaminated with blood and body fluid but not blood or body fluid exposure to non intact skin.

False , both are considered . As well as splash to eyes, nose, or mouth.

Which of the following is false? - All Medicaid programs administered by the state in partnership with the federal government - Some states also provide coverage for pregnant women - CHIP provides financial assistance for children of families that do not qualify for Medicaid ◦Federal pays annually (capitation) as well as fee-for-service basis

False for "Federal pays monthly (capitation) as well as fee-for-service basis" Medicaid Manage Care Organization ◦State pays monthly (capitation) as well as fee-for-service basis ◦Uses primary care physicians as gatekeepers

T/F medicare cover hearing aids, routine hearing exams, or fittings for hearing aids

False! Note: Original Medicare and most Medicare Supplement (or Medigap) ◦Don't cover hearing aids, routine hearing exams, or fittings for hearing aids

True or false, state and federal governments listed optional that all hospital patients in New York State be given certain information and materials when admitted to a hospital

False! It is required! - NYS Pateint rights Information provided separated into 2 sections. ◦Information on how to best get assistance ◦Explains patient rights

T/F, patient with DNR can not be intubated.

False!!!! *Patient with DNR, they are NOT DNI! If a patient heart is fine, and O2 is low, you still can intubate.

T/F Both quality assurance and quality measurement looks for issue in structure.

False, QA looks at individual whereas QM look issue in structure, process and outcome.

T/F, quality measurement result can decribe observed level of activity (eg. immunization rate, rate of parental satisfaction with referrals) and why the level is as it is.

False, does not tell WHY! }Results cannot reveal which factors account for differences in measured levels of quality. }Answers to such questions require additional investigation and may serve as the starting point for program management initiatives or quality improvement efforts.

T/F you need medicaid as well as medicare to qualify for medicare advantage plan.

False, only medicare is required but medicaid do helps to cover the premium of the part B through medicare advantage plan.

T/F palliative care is hospice care when a patient has > 6 months of life.

False, palliative care is hospice care when life is < 6 months. If > 6 months, just palliative care. - Some think of palliative care as a hospice that isn't defined by the 6-month hospice benefit. - Palliative care programs don't have to provide the same range of core services as hospice care - such as bereavement, spiritual, social work, patient/family, etc. - All hospice is palliative, all palliative isn't hospice and doesn't have to provide the same range of services.

T/F, discharge paper needs to be provided when is patient transferring from hospital to hospital?

False, transfer of information [transfer paper work] rather than discharge paper needed to be provided to next facility for care continuity.

T/F medigap is replacement of medicare.

False. Only pays after medicare plan A and B paid. fills in the GAP }Medigap is Medicare Supplement Insurance that helps fill "gaps" in original Medicare and is sold by private companies. }A Medicare Supplement Insurance (Medigap) policy can help pay some of the remaining health care costs, like: ◦Copayments ◦Coinsurances ◦Deductibles }You must have Medicare Parts A and B to qualify for Medigap }Claims are paid by Medicare first, then Medigap covers the remaining balance } }Medigap policy is different from a Medicare Advantage Plan. ◦Advantage plans are ways to get Medicare benefits, while a Medigap policy only supplements your Original Medicare benefits }

Which level is HIPAA Privacy Rule regulated? - State - Federak

Federal -**The HIPAA Privacy Rule is a federal regulation. The Rule ◦Sets standards for patient privacy and confidentiality. ◦Sets severe civil and criminal penalties for people who violate a patient's privacy.

What are the key aspect of PA education in Maturation Period? ◦Curriculum development

Federal government involvement ◦Prompted changes to reflect educating Primary care providers ◦Provide incentives/grants PA program included health promotion and disease prevention topics ◦Improve student knowledge and skills in specific populations Aging, racial and ethnic minorities and those affected by HIV

What is the FQHC program?

Federally Qualified Health Centers -Medicare and Medicaid can reimburse higher for rural, underserved areas if clinic is non-profit

Feldshers were seen as _________________ to physicians whereas Barefoot doctors seen as physician ______________

Feldsher = complementary barefoot = substitute

In pioneer period, what are the student attributes?

In the 1970s ◦Most states did not have laws governing PA practice Students were risk-takers ◦No guarantee they can practice after graduation **After graduation ◦Many went to establish national PA organizations that address professional issues Some of these first PAs became educators for future PAs

What is Credentialing?

In-depth background check (including criminal background check) through Credentialing specialist Will verify: Previous employment References Malpractice history (even as a student) Education records, Licenses, Certifications May take a while because credentialing board - usually meet every month or few months

What is other MCO Plans

Indemnity Insurance (aka: fee-for-service). ◦Almost nonexistent currently ◦Regular premium paid by the enrollee to Ins co. ◦Protects beneficiary from financial costs of healthcare ◦Coverage initially only for illness (Catastrophic) ◦Company-paid what they felt was a maximum appropriate charge, enrollee had to pay anything not covered by insurance. ◦No cap on how much providers can bill. - No fee schedule, if provider bill $400 for physical, insurance cover 200, member pays the rest.

What is the ARC-PA? What doe it protect?

Independent accrediting body that provides accreditation to all PA programs within the US Protects the interest of PAs and the public by making sure PA programs are well prepared and compliant

What is an essential feature for NPs?

Independent practice NPs are advance degrees as justification for independent practice (now DNP) ◦Strong lobbying

Why are PA students at higher risk of needle stick and shapr injury?

Inexperience

Special interest group

Informal groups composed of individuals sharing a common interest Example of SIGs: American Academy of Physician Assistants in Legal Medicine Jewish Association of Physician Assistants Physician Assistants in Research

What are other models for PA development?

Initial Years: 1965-1971 - 16 programs formalized PA/NP education - All initially used the Duke training model - Emphasized inpatient medicine and surgery [easier for direct supervision] - used academic centers as training facilities to coincide with medical students

Where did Dr. Eugene stead received his funding for Duke's program?

Initially from ▫Federal Government's National Heart Institute ▫Funding from: National Center for Health Services Research and Development ▫Federal funding Increased with: Wider acceptance of PAs Ease of training and deployment to underserved areas

Who are the educators of PA education in Mid 1960s through 1970s?

Initially, none were PAs Many were clinicians and Educators and came from a variety of backgrounds such as: ◦Medicine, social work, nursing, biology and chemistry

leadership style: transformational

Inspires employee to have a shared vision distills values and hopes and needs of followers into a vision.

What may institution ask after getting hired?

Institutions (Eg. Hospitals) ◦Credentialing - Process takes a longer time ◦Malpractice usually paid for by the facility ◦EMR training ◦Any other related training - eg. HIPAA ◦May ask for procedure logs (keep detail documentation - Typhon; portfolio

What does insurance do?

Insurance ◦Company that pays medical expenses ◦Privately funded by Employers Consumers ◦Publicly funded - eg. Medicaid ◦State government regulates: Cost to consumer or employer Benefits covered Privacy

What is I-Stop?

Internet System for Tracking Over-Prescribing (Prescription Monitoring Program)

What is I-Stop system?

Internet System for Tracking Over-Prescribing ◦- Prescription Monitoring Program - Check I-STOP when writing prescriptions for Schedule II, III, and IV controlled substances - REQUIRED!

Potential Job?

Interview ◦With HR ◦Also with department ◦May ask clinical questions Salary Negotiation ◦Can ask in general terms, what is the range of salary for this job. ◦Look up what others are being paid in the field and ask for similar pay ◦You may be asked by HR for an approximate salary that you are looking for - Always ask for higher ◦Negotiation begins after you are offered the job. Can ask about benefits at HR interview

What are Feldshers?

It is a European concept in the 17th and 18th Centuries that was introduced into the Russian Military What it did: trained military personnel basic medicine

PA education key points

KEY POINTS } PA education is based on the medical education model of general medical didactic knowledge first, followed by application through full-time clinical rotations. Although there have been several successful variations on the medical education model, high-speed, high-volume medical education has been a hallmark of PA education. The PA profession's ability to work in many specialties and settings rests on the breadth and depth of the PA curriculum. Early PA education leaders came from many professions. Eventually, PAs themselves became the leaders of PA programs and created stability within PA programs that were not initially seen in the early days of PA education. Since then, PA education has undergone several periods of rapid growth in the number of programs.

What are four types of Physical Exam?

Know that higher level want to bill, the more components [all have to be pertinent] A. Problem focused [ 1-5 element from same system] B. Expansed focused [ 6 - 11 element from same system] C. Detailed [At least 12 elements from 2 or more area or at least 2 elements from 6 area] D. Comprehensive - at least 2 elements each from 9 areas.

What are AAPA Values 2016-2020 [4]?

Leadership and service unity and teamwork accountability and transparency excellence and equity LUEA

What is the most powerful combination of guidance of care?

Living Will with Health Care Proxy }Powerful combination of documents to guide care }Living will clearly outline what the patient wants }Health Care Proxy can guide care based on situations

What do chapter organizations provide?

Locally based CME, networking opportunities, timely information regarding advocacy and job resources

What are PA-Specific Reimbursement?

Medical and surgical services delivered by PAs are covered by most third-party insurances - PAs must have an NPI number - Must Submit a Medicare application form (855i) Third-party insurances include: ◦Government-funded (Medicare, Medicaid, TRICARE) Tricare - provide services for military personnel ◦Private payers/insurance companies - Each payer has its own guidelines regarding service coverage and payment.

What is the third leading cause of death in the US?

Medical error }10% of US deaths are now due to medical errors

What are types of government insurances?

Medicare Medicaid Federal Employees Health Benefit Program ◦Government pays for medical expenses unless paid by private health plans ◦Funded by taxpayers

What is medication reconciliation?

Medication reconciliation is the process of comparing a patient's medication orders to all of the medications that the patient has been taking.

What is the MEDEX model?

Medicine Extension model

What is limited permits of PA?

Meets all the above characters [21 age, good moral character, completed approved program], has all requirement met but not ready to get NCCPA exam yet. Use limited permit ◦Given to persons who fulfill all requirements for licensing except that related to the examination Limited permittees needs direct supervision of a licensed physician Expires one year from the date of issuance Can be extended for one year if endorsed by current or prospective supervising physician (maximum time a limited permit can be held is two years)

Who are the Students of PA education in Mid 1960s through 1970s?

Military corpsmen Also attracted allied health professionals ◦Nurses, Medical Technologist, Radiology/Respiratory therapists and other health related professionals Students with no healthcare experiences were also admitted

What is TRICARE specific for?

Military personnel

What are AAPA's Missions and Visions of 2016-2020

Mission: Advance members' careers and patient health Vision: Transform health through patient-centered and team-based practices

Who are educators in maturation period?

More PAs now. ◦Majority from the pioneer programs All PA program had at least one PA on staff ◦Eventually, the majority of PA program directors were PAs. Physician and other clinicians continued to serve as guest lecturers - Teaching style was less traditionally academic and more clinical with Student-centered faculty practice

What is a MOLST form?

More specific form of DNR - Medical Orders for Life Sustaining Treatment (MOLST) The MOLST form is a set of medical orders for patients with advanced illness who might die within 1-2 years; require long-term care services; or wish to avoid and/or receive specific life-sustaining treatments now.

Current (Since 2010) & The Future What EDUCATION STATUS are needed for current PA graduates?

Most Programs average 26 months Increase number of prior healthcare experience required Courses taken Basic sciences behavioral sciences and clinical medicine across subjects such as anatomy, pharmacology, microbiology, physiology and more Complete at least 2000 clinical hours in: Family medicine internal medicine Ob/Gyn pediatrics general surgery emergency medicine psychiatry

leadership vs. management

NOT the same thing, complementary to each other -management: the main function is to PRODUCE ORDER AND CONSISTENCY through processes, such as planning, budgeting, organizing, staffing, and problem-solving -leadership: main function is to PRODUCE movement and CONSTRUCTIVE OR ADAPTIVE CHANGE through processes such as establishing direction through visioning, aligning people, motivating, and inspiring

What is National Provider Identification Number (NPI)?

NPI is a unique identification number for you. ◦Developed by Center for Medicare and Medicaid Services (CMS) ◦Purpose is to improve the efficiency and effectiveness of the electronic transmission of health information }NPI number usually emailed immediately to you.

What is the NY chapter organization?

NYSSPA (New York State Society of PAs)

NCCPA

National Commission on Certification of PAs An independent certifying organization issues PANCE and PANRE and monitors CMEs

how to qualify medigap?

Need part A and Part B to qualify.

What is Principle Diagnosis?

Need to indicate the Principal Diagnosis "that condition established after study to be chiefly responsible for occasioning the admission to the hospital for care" Ex: Pt admitted for abdominal pain Workup -> gastric ulcer Final DX: Gastric Ulcer = Principal Dx Reason keeping patient to the hospital for care If unable to find a principal diagnosis, can use abdominal pain for principal diagnosis.

What is the most common efficient method of transmitting blood-borne pathogens between patients and clinicians?

Needlestick and sharps injury

What procedures are further consent required ?

Needs additional consent if is following procedure ◦Surgical procedures ◦Blood transfusions ◦Central line placement ◦Radiologic studies with contrast media CT MRI ◦Thoracentesis/paracentesis* ◦Lumbar puncture

What is the definition of Negligence?

Negligence - whether the care provided failed to meet the standard of care reasonably expected of an average physician qualified to take care of the patient in question.

What is following statement describing? ◦Group of physicians and/or medical facilities associated with the same managed care plan A. Capitation B. Fee for service C. Explanation of benefit D. Network

Network

Which regulatory body is responsible for the general supervision of all educational activities within the State?

New York Board of Regents NY Board of Regents presides over The University and the New York State Education Department

May 2017, last seen in office, never came back and want to see a physician in office, establish or new patient?

New patient since 3 year has past.`

Inpatient Time-Based Coding

No need specific

Outpatient Time-Based Coding

No need specific

What information could link back to the patient? Protected Health Information

No need to memorize

Should you still go to work if you have infectious disease?

No, dude!. }Example: ◦Infectious diarrhea (gastroenteritis) ◦Fever (fever & cough or sore throat during flu season) ◦Rash - due to chicken pox, herpes zoster, scabies

Does absence of disease mean people are healthy?

No.

Are Family members, friends or relatives, good sources of interpretation?

No...but in case of emergency, it is okay. Family members may become emotionally distressed at receiving upsetting medical news and may incorrectly interpret information or become unable to continue interpreting. A qualified medical interpreter can more impartially relate sensitive information, often with better judgment and bedside manner

Is chart co-signature required PRIOR to execution of an order placed by an PA?

No; co-signature not required prior to execution of an order (especially inpatient)

Are PAs allowed to perform an x-ray?

No; only a certified radiologist can

Which three specialty areas are now separated as special assistants?

Orthopedics, Radiology, or Urology

Where are areas considered as specialist assistant?

Orthopedics, Radiology, or Urology Specialist assistant is not the same as PA. They are trained under specialist assistant school.

What are some other influences on Health disparities? ~ for better [7]

Other influences on health include the availability of and access to: ◦High-quality education ◦Nutritious food ◦Decent and safe housing ◦Affordable, reliable public transportation ◦Culturally sensitive health care providers ◦Health insurance ◦Clean water and non-polluted air

How long has NYSSPA existed?

Over 35 years

Is PA - Physician Relationship supervision or collaboration?

PA - Physician Relationship is collaborative and collegial - (Clinical Colleagues) Supervision does NOT mean: A physician must be physically present Should direct every aspect of the care PAs has a high degree of autonomy and has physician avaliable for consultation.

Why is PA and physician collaboration effective?

PA - Physician has historical ties, a similar training scheme, and the PA profession's commitment to collaboration and team-based practice.

What are the safety issues related to patient, clinician, PA students?

PA students or PA }Needlestick and Sharps Injury }Student Mistreatment

Who developed CASPA and when?

PAEA in 2001

Who endorsed the Master's in PA and when?

PAEA in 2009

who are EDUCATORS in The 2000s - Moving forward?

PAs Challenges ◦To retain PA faculty and equip them with the credentials and skills to have rewarding career in academia

What are the Professional Behavior PA should have toward other Professionals?

PAs are expected to demonstrate professional relationships with physician supervisors and other health care providers

What can PAs do? [5]

PAs can: 1Perform physical examinations 2Diagnose and treat illnesses Prescribe medications 3Order and interpret lab tests 4Perform procedures 5Assist in surgery and enhance healthcare coordination

What are TYPES OF ACCREDITATION?

Provisional - New program and valid for 5 years after matriculation of first cohort. Continued - [Probation to continued ] - Compliance with standards - Continue to Reaccreditation (continued) -Remains in effect until the next accreditation cycle, the program closes, withdrawn, or fails to comply with the standards Clinical Postgraduate Program - voluntary Probation - In compliance with standards Temporary status Valid for 2 years Quality of education to students is threatened [may lead to accreditation withdrawn] Administrative Probation - Temporary status when program has not complied with administrate requirements [ Such as pay fees, submit reports ] Withheld - 30 day appeal process - Program not in compliance with standards - Can proceed to withdrawn status Withdrawn - Program not in compliance with standards - Not capable of providing acceptable education to its students - Failure to comply with accreditation requirements Voluntary inactive status - ◦Programs decision to suspend its operation

What is public health definition?

Public health promotes and protects the health of people and the communities where they live, learn, work and play (www.apha.org)

What is Public Health System?

Public health systems - commonly defined as ◦"all public, private, and voluntary entities that contribute to the delivery of essential public health services within a jurisdiction." ◦Can also be described as a network of entities with different roles, relationships, and interactions that contribute to the public health and well-being within a community.

What are some highlights of PAEA?

Publishes Journal of Physician Assistant Education (JPAE) Supports research and grants for PA education Developed the Central Application Service for PA (CASPA) - 2001 Endorse the Master's degree as entry level and terminal degree for PA education -2009

Where is the only place that does not authorize PAs to practice medicine including medication prescribing?

Puerto Rico All states, the District of Columbia, and all U.S. territories except Puerto Rico: Authorize PAs to practice medicine, including prescribing medications.

To make everyone healthy include? What are purpose of Public Health?

Purpose of Public Health ◦Prevent injuries ◦Promote and encourage healthy behaviors ◦Assure the quality and accessibility of services ◦Prevent epidemics and spread of disease ◦Protect against environmental hazards ◦Respond to disasters and assist communities in recovery

Which group does Quality assurance measures focus on?

QA look at compliance against certain necessary standards with focus on individual rather than the structure of facility }It refers to activities and programs intended to "assure" or promise improvement in the quality of care in a defined medical setting or program.

Questions PAs should ask What would happen if there is physician on staff that do not with to work with PAs?

Questions PAs should ask Are there physicians on staff who may not want to work with PAs? This could signal trouble ahead. Physicians in a group setting need to agree about hiring a PA and be open to teaming with you. All it takes is one bad apple who resents PAs to undermine collegiality, teamwork, and your happiness at a job.

YORK PA PROGRAM HISTORY

Re-Accreditation visit was November 30 and December 1 - 2017. ◦March 2018 - York received a maximum of 10 years for continued accreditation Next Site visit - March 2028 but still need to submit continuing self study report ◦York's Complete Accreditation History http://www.arc-pa.org/wp-content/uploads/2020/08/Accreditation-History-York-College-164.pdf

When did funding resume/increased again for PA program?

Reauthorization for funds were delayed until the passage of the 2010 health reform [Affordable Care Act [Obamacare]]

History - Regarding to level of complexity - know this.

Regarding to level of complexity HPI ~ OLD CARTS ~ how many components used [1-3, 1-3, >4, >4] ROS ~ how many systems reviewed [0, 1, 2-9, > 9] PMSH - How many involved. [0, 0, 1/3, All of 3] There are four levels of history

What is the Good Samaritan Law?

Rendering voluntary emergency care (cannot be legal obligation or job to do so or if you caused problem) Good Samaritan laws are written to encourage bystanders to get involved in these and other emergency situations without fear that they will be sued if their actions inadvertently contribute to a person's injury or death.

◦Identify the allied health professional below: Need a blood gas, Nebulizer treatments

Respiratory Therapist

What if you have to Disclose PHI?

Review HIPAA guidelines for allowed disclosures of protected health information.

Which PA leadership style best fits physician assistants?

SERVANT LEADERSHIP coined by Robert K. Greenleaf Servant-leader is one who is a servant first ◦A long-term, transformational approach that has the potential for creating positive change throughout our society. Note: Transformational approach: distills values and hopes and needs of followers into a vision.

What are statute of NY licensure and how to obtain them?

STATUTE ◦New York Education Law ◦New York Public Health Law In NYS, the practice of PA work requires NYS licensing. At the time of application: ◦At least 21 years of age ◦Is of good moral character ◦ ◦Has successfully completed a minimum of a bachelor's degree (or equivalent) ◦Has completed an approved program Includes at least 32 credit hours of classroom work 40 weeks (*1600 hours) of supervised clinical training ◦Has obtained a passing score on an examination acceptable to the department (exam used is the NCCPA examination)

What is roles of PAEA?

Serve as a resource for those interested in educational aspects of the PA profession collects and publish info on PA programs provides representation to affiliated organizations in health education, health care policy, and the national certification of PA graduates ensures quality PA education through the development and distribution of educational services and products specifically geared toward meeting the emerging needs of PA programs, the PA profession, and the health care industry

Exception to 85% coverage. When can you get 100% coverage?

Services provided in office and clinics (outpatient services) }Billed under "Incident to" provision ◦Billed under the Physician's NPI ◦Billed at 100% coverage rate

What is exception to 85% in hospital setting?

Services provided in-hospital setting (hospital inpatient; hospital outpatient and ER) }Billed under "Shared or split Visit" rules ◦Billed under the Physician's NPI ◦Billed at 100% coverage rate ◦Only recognized by Medicare

Key feature of PA educationMid 1960s through 1970s Diversity of Education What avoids the adopt identical curricula?

Several factors guard against this possibility of "sameness" ◦First set of accrediting standards (1971) - Firmly established competency-based approach and promotes diverse educational approaches ending in same core principles. - Teaching students the following standards while ensuring students learning what they learning by multiple ways of teaching! - Promotion of diverse educational approaches among programs ◦Sponsoring institutions such as hospitals, colleges, universities

What are the benefit of performance improvement?

Similar to quality improvement. }Ensure patients receive the appropriate and best possible care }Ensure satisfactory experience for patients and their families }Ensure adherence to evidence based practices }Reduce or avoid negative outcomes }Comply to with regulatory and accrediting standards }Provide efficient and effective therapies }Maintain a recognized standing in the community

What is Limitations on Eligibility for PANCE? How many time can you take PANCE?

Six-year, six attempt rule ◦Eligible to take PANCE for up to six years after program completing ◦During that six-year period, PANCE may be taken a total of six times. ◦When either the six attempts or six years are exhausted, whichever occurs sooner, the individual loses eligibility to take PANCE. ◦The only way to establish new eligibility is to complete a full ARC-PA accredited physician assistant educational program again.

Why do you need to know about cultural competency in relate with quality of healthcare?

So you can understand the concept of cultural competency so as to effectively deliver quality healthcare. E.g. Cupping ~ abuse vs therapeutic?

◦Identify the allied health professional below: Needs services for patients at home

Social Worker or Care Managers

leadership style: sharing power.

Some leaders choose to exercise at least some leadership through the other stakeholders in the organization. ◦Give up some personal power in return for more ownership of decisions making process and goals of the organization

What is Do Not Intubate (DNI)?

Specific will for not receiving intubation while chest compressions and cardiac drugs may be used.

What are Standards of ARC-PA?

Standards are used for: ◦Development [establishment of program] ◦Evaluation [approved program continuing accreditation] ◦Self-analysis of PA programs [continuing self study]

What are Health Insurance Portability and Accountability Act (HIPAA)?

Started in 1996 and divided into two major parts -Title I ◦Deals with insurance coverage for workers and families -Title II ◦Deals with Rules and Regulations regarding healthcare ◦3 components - (Know as the "administrative simplification" provisions of HIPAA 1. Electronic exchanges 2. Security 3. Privacy

Identify below as an example of structure, process, or outcome 1. ◦Percentage of patients with asthma for whom appropriate medications are ordered. 2. ◦Test results within a range indicating effective functioning (eg. Appropriate DM Tx) 3. ◦Number of nurses per patient. 4. ◦Percentage of physicians who are board-certified. 5. ◦Presence of quality improvement programs. 6. ◦Number of times adolescents are provided guidance on smoking avoidance. 7. ◦Number of avoidable complications and deaths. 8. ◦The experience with health care and/or the level of satisfaction with care. 9. ◦Percentage of females of specified age receiving mammography. 10. ◦Number of patients successfully treated.

Structure 3. ◦Number of nurses per patient. 4. ◦Percentage of physicians who are board certified. 5 ◦Presence of quality improvement programs. Process 9. ◦Percentage of females of specified age receiving mammography. 1. ◦Percentage of patients with asthma for whom appropriate medications are ordered. 6. ◦Number of times adolescents are provided guidance on smoking avoidance. Outcome 10. ◦Number of patients successfully treated. 2. ◦Test results within a range indicating effective functioning (eg. Appropriate DM Tx) 7. ◦Number of avoidable complications and deaths. 8. ◦The experience with health care and/or the level of satisfaction with care.

What is inter-professional education? [IPE]

Students from two or more professions learn about, from, and with each other to enable effective collaboration and improve health outcomes. ◦Example: PA, Nursing, CLS Students

What is In NYS - Prescriptive privileges Outpatient settings? clinic, private practice, HMO

Supervising physician delegates to the PA the writing of prescriptions for both - non-controlled substances, e.g. antibiotics, antihypertensive meds - controlled substances Schedule II-V

What is TRICARE Reimbursement for PAs?

TRICARE: ◦Covers all medically necessary services provided by a PA. ◦The supervising physician must participate in TRICARE Reimbursement for services provided by PAs under TRICARE fee-for-service 85 % of the allowable fee for comparable services rendered by a physician in a similar location 65% of physician's allowable fee for assisting at surgery for comparable services.

What does PDSA cycle used for?

Testing a change

Where does the AAPA want programs to be developed?

The AAPA urged to APAP (now called the PAEA) to develop new programs in areas/states where programs were not available

Why is there a continued debate regarding the development of new programs?

The PA profession continues to debate regarding the development of new programs BECAUSE: It is impossible to make a prediction about the future healthcare workforce

What are the Scope of Practice of PA determined by?

The PA's education and experience State law Facility policy and the needs of patients.

What is the definition of health care quality?

The degree to which health care services for individuals and populations increase the likelihood of desired health outcomes and are consistent with current professional knowledge.

What are the key aspect of PA education in Maturation Period?

The demand and growth for PAs resulted in educational changes ◦PAs in different setting and specialty Several trends were Identified ◦Acceptance of PA programs ◦Curriculum development ◦Postgraduate programs ◦Issues of Standardization

What are feldshers?

Trained military personnel basic medicine Introduced into the Russian Military Provided and Improved healthcare - On the battlefield - Remote Russian villages - Remote parts of Alaska - Continue practicing even after their retirement

What is laterality of ICD10?

The left or right side of the body.

Information of Accreditation Cycle How many years are the maximum length of time between comprehensive evaluations? ◦Established PA programs is ____ years. ◦Established clinical postgraduate PA programs is ____ years. ◦Program completing Probation status is ____ years. ◦Clinical postgraduate PA program on probation is ____ years.

The maximum length of time between comprehensive evaluations ◦Established PA programs is 10 years. ◦Established clinical postgraduate PA programs is 6 years. ◦Program completing Probation status is 8 years. ◦Clinical postgraduate PA program on probation is 3 years.

What are Barefoot Doctors?

The role of Barefoot Doctors arose during China's Cultural Revolution in 1965; barefoot doctors were to act as physician substitutes but works closely with community hospital for training and supervision - Barefoot Doctors were brought up in response to Chairman Mao's directive to reorganize healthcare

What does federal funded facility must provide for pt?

Title VI of the Civil Rights Act of 1964 ◦Federally funded health facilities must provide interpretation for all patients who request it.

Medical Decision-Making What is needed to meet a specific level for established patients?

To meet a specific level for established patients, have to have 2 of the 3 elements at least at the same level

What did the responsibilities of a Feldsher include?

They had privileges in rural setting - diagnose and prescribe - institute emergency treatment - provided a "complementary" not "substitutional" role to physicians - concern for second-class healthcare led to greater supervision

What training did Feldshers recieve?

They received formal training in the late 19th century - taught in the same institution as nursing school - two year program - by 1913- graduated 30,000 Feldshers (equal # of graduates compared to physicians; 90% being female)

How were Barefoot Doctors trained?

They were chosen from rural production brigades (they were farmers with no background in medicine) - 2-3 month training courses in regional hospitals and health centers - barefoot doctors were designed to function independently but linked to community hospital for training and supervision

Third-party insurances include?

Third-party insurances include: ◦Government-funded (Medicare, Medicaid, TRICARE) Tricare - provide services for military personnel ◦Private payers/insurance companies

What is explanation of benefits (EOBs)?

This is not a bile ◦Explanation of charges after evaluated by insurance company - what insurance paid and what pt pays and how much provider charged.

What are Schedule I- Limited medical use in the US?

Used in research primarily* -High abuse, questionable clinical application -Marijuana - medical marijuana with increased usage Ecstasy In NYS, PA can prescribe Marijuana after completion of approved NYSDOH course. ◦New York State Medical Marijuana Program PAs can prescribe Marijuana in NYS After completion of the approved NYSDOH course Registered as a approved practitioner

How does HMO receive payment?

Uses capitation or fee-for-service depending on plan enrolled Capitation - Fixed payment for services per-member per-month [Whether seeing pt or not]

What is allied health?

Usually needs a license or state registration to practice Association of School of Allied Health (ASAHP) defines allied health as: } "Those health professions that are distinct from medicine and nursing." "Allied health encompasses a broad group of health professionals who use scientific principles and evidence-based practice for the diagnosis, evaluation and treatment of acute and chronic diseases; promote disease prevention and wellness for optimum health, and apply administration and management skills to support health care systems in a variety of settings." ◦Paramedic, EMT ◦Clinical Laboratory Technologist/ Medical Tech ◦Medical Assistants ◦Physical Therapists ◦Respiratory therapists ◦Occupational therapists ◦Audiologist ◦Radiology tech ◦Etc., Etc.,.............

What makes this 99202 rather than 99201? Example: 99202 Expanded problem focused Hx Expanded problem focused PE Straightforward decision making. New pt. visit 42-year-old female with a history and rash consistent with poison ivy who is not responding to over-the-counter medication. Requires: Review of Hx - expanded prob focused Expanded prob.-focused exam Form Dx and Tx plan - straight forward Discuss Dx and Tx options and prevention Write Rx

What makes this 99202 rather than 99201? All are at same level but not simpler.

What does it mean by protective social factor?

What protect your health? }Protective Social factors include: ◦Social Support [family tell you to get check up -> promote health] ◦Self Esteem [high SE -> prom. change making] ◦Self efficacy [your ability to organize and make change] Self efficacy - capabilities to organize and execute the courses of action required to manage prospective situations." Self-efficacy is a person's belief in his or her ability to succeed in a particular situation.

If pt does not have documentation for the advanced directives, what happens then?

When lacking advance directives, NYS will prioritize decision making in this order ◦Spouse or domestic partner ◦Son or daughter 18 or older Oldest with priority ◦Parent ◦Brother or sister 18 or older ◦Close friend With documentation anyone can abstain from surrogacy, priority then falls to next member on list

What is inter-professional collaboration? [IPC]

When multiple healthcare workers from different professional backgrounds work together with patients, families, caregivers, and communities to deliver the highest quality of care. ◦Example: PA, RN, Social worker, etc.

What to do when unsure of patient's capacity for informed decision?

When unsure ~ can seek evaluation by psychiatry to determine the capacity.

Which university has doctorate of PA program?

doctoral programs in PA Baylor university Clinical Doctorates - US army and Baylor University ◦General Surgery/Intensivist Physician Assistant Residency and Doctoral Program ◦18-month program of subspecialty rotations, formal didactic instruction, and clinical research

What are schedule II drugs?

drugs that have medical use. High potential potential for severe psychic or physical dependence/abuse Tightly controlled prescribing (no refills) Ex. Morphine, Methamphetamine, Hydromorphone (Dilaudid)

How often are PAs required to complete infection control practices training?

every four years

How were PAs tested before 1974?

exam issued by NBME had 880 candidates in Dec 1973

What components are included in Personal Protective Equipment (PPE)?

gloves, gowns, masks, protective eyewear`

Describe the current trends in practice settings

Whose the majority? White, 30-39 age, 5 - 9 yrs of clinical experience, female > male

does leadership styles change over time? What promote the change?

YES! ADAPT to situation -Leadership style should be adapted to - the requirements of the people involved - the demands of the situation - the challenges facing the organization

Was upward mobility encouraged for Barefoot Doctors?

Yes - given priority for admission to medical school - approx 1/3 of Chinese medical students were barefoot doctors

Patient with DNR, can they still be intubated?

Yes, DNR only abstain providers from perfoming CPR.

Can New York State PAs prescribe medical marijuana?

Yes; after completion of the approved NYSDOH course and registered as an approved practitioner

Are PAs authorized to certify a disability for the purpose of obtaining a disability parking permit?

Yes; but don't go out giving everyone a parking permit

You are a PA practicing in emergency medicine - was driving home from work after an overnight shift, saw an accident on the road and stopped to help. You did everything you could to help the patient including performing CPR - by the time EMS arrived, the patient had regained consciousness Patient was taken to the hospital and survived, but with multiple disabilities Few months later, the patient decides to sue you, since she had a DNR in place and didn't want to be resuscitated. What would you do?

You are covered by the Good Samaritan Act Contact the legal team where you work to handle this situation

Do you need medicare for medicare advantage plans?

You need medicare but does not need medicaid. Can still have medicare with private insurance with premium.

Who is considered as "second victim"

You! Second victims are "healthcare providers who are involved in an unanticipated adverse patient event, medical error and/or a patient-related injury and become victimized in the sense that the provider is traumatized by the event." The reaction has been liked to an acute stress disorder Embarrassment Denial and detachment Loss of confidence Inappropriately adjusting practice Emotional and physical health - Depression - Anxiety - Re-experiencing the event - Substance abuse

leadership style: collaborative

a collaborative leader tries to involve everyone in the organization in leadership, initiates discussions, pinpoints problems, and keeps track of the organization as a whole

leadership style: democratic

a democratic leader looks at his and others' positions in terms of responsibilities rather than status and often consults in decision-making

Subspecialty orgs are liaison to AAPA to

address challenges such as reimbursement, clinical practice, and regulation.

What concepts are professionalism associated with?

altruism, accountability, respect, integrity, ethic[ism], lifelong learn[ing], honesty, compassion, excellence, self-regulating, service

leadership style: autocratic

autocratic leaders insist on doing it themselves, they have all of the power, make all of the decisions and don't often tell anyone else about what they are doing

Where does the trust that form therapeutic relationship build upon?

builds from the belief that the clinician possesses expert knowledge (that will be applied to the benefit of individuals and society) and will avoid self-interest while acting on behalf of those served.

What are the challenges in leadership?

challenges can arise from - external/internal sources - leadership itself

leadership style: persuasion

convinces people through argument, reasoning, selling techniques that what the leader wants is in fact the best course or in line with what they want to do

What is the supervisory ratios for NYS prior to March 2013 for inpatient? what about after March 2013?

inpatient (hospital, LTC) prior to 2013: 6 PA to 1 physician after 2013: 6 PA to 1 physician

Why does Physician's Assistant and Physicians' assistant not work?

it implies possession - there should be NO apostrophe HOWEVER, Apostrophe was used in INITIAL years of AAPA development

Key feature of PA education Mid 1960s through 1970s What is the benefit of Compact Curriculum?

keep cost down - 75% of medical school in 50% of the time - ◦Foundation in arts and science [e.g. preventive medicine] - Student assessment [focus on outcome] - Each course was rigorously examined [Each course build on another] to reduce redundancies and maintain practicality. - Focus on case-based scenarios 1. Teaching contributed by community participation physicians and clinical rotations 2. Uses standardized pts. 3. Instructors from a wide variety of disciplines.

leadership style: charisma

leaders pull others along by the power of their personalities alone

leadership style: transactional

leadership based on transactions btw leader and followers, followers motivated in an exchange relationship by internal and external rewards to accomplish the goal

What is schedule I drugs?

limited medical use in the US Ex. Marijuana, Ecstasy high abuse potential

What is the legislation for? Is it federal or state specific?

necessary to safeguard you and your patients }Legislation are State Specific** }It allows you the ability to provide the highest level of care to your patients In NYS, legislation, as it relates to PA, are: Delegatory [at practice level] and regulatory [by state]

Is there a limit on the number of supervising physicians that may be assigned to one PA?

no; there is no limit

Are hospitals required to hire PAs?

no; ◦Hospitals are not required to employ PAs

When was NCCPA established?

not-for-profit in 1974

Which are the three area that shared visit concept do not apply to?

procedures critical care services nursing home visits.

What is non-maleficence?

protecting the patient or others from harm DO NO HARM.

What is a DEA number?

registered number assigned to clinicians by the Drug Enforcement Administration Allow prescription of controlled substances (narcotics)

What are NYSSPA achievements?

removal of 24-hour countersignature prescribe Schedule II-V, protected rights to first assist in surgery, awarded 60k in scholarships, supported appointment of a PA rep on the office of professional misconduct board

Learning Objectives

}Define Quality and Recall measures of quality of care }Define Quality Assurance and Recall examples of Quality Assurance activities }Define Quality Improvement and Recall examples of Quality Improvement programs

leadership style: the great man or woman

one who is BORN TO LEAD, due to innate qualities of intelligence and personality leading all to subordinate themselves to him or her

DO NOT USE LIST 1

outlines the most common misread abbreviations

DO NOT USE LIST 2

outlines the most common misread abbreviations

What is the supervisory ratios for NYS prior to March 2013 for outpatient? what about after March 2013?

outpatient prior to 2013: 2 PA to 1 physician after 2013: 4 PA to 1 physician

What are responsibilities of a Health Care Proxy?

responsibilities of a Health Care Proxy - Decisions related to artificial nutrition and hydration ◦specifically, tell your wishes to your agent or write your wishes in your Health Care Proxy form. - Decide whether or not CPR should be used -------------------------------------------------------------- - Once the agent's authority begins, he or she has the right to get your medical information and records to make informed health care decisions for you.

How is a continuous improvement process such as Quality improvement achieved by?

review, critique, and implement positive change to achieve quality improvement ALSO range of practices geared toward improving performance

What schedule drugs are Valium, Xanax, Ativan, Versed, Klonopin?

schedule IV Lower abuse, efficacious

Benefits of NYSSPA

scholarships, medical jeopardy, CMEs, conferences

With Feldshers, concern for ______________ healthcare led to greater supervision

second-class

What are goals and purpose of NYSSPA?

support the profession, participate in CEs involve in professional and legislative advocacy improve awareness of PA roles encourage a mutually beneficial relationship between PA and Physician

What is Another Option alternative for PANRE?

the "Alternative to PANRE" - A Pilot program - Eligible if due for recertification in 2018 or 2019 - Answer 200 questions over 2 years - focus on core medical knowledge (basic knowledge that every PA should know regardless of specialty) **25 Qs each quarter [every 3 months]. Benefit: Less stressful.

What is Cultural Competency?

the ability to understand, appreciate, and interact with persons from cultures and/or belief systems other than one's own, based on various factors - not just your own culture .

What is cultural competency?

the ability to understand, appreciate, and interact with persons from cultures and/or belief systems other than one's own, based on various factors.

leadership style: follower-turned-leader

the best LEADER RISES FROM THE EXPERIENCE OF BEING A FOLLOWER

What is Culture?

the combination of a body of knowledge, a body of belief and a body of behavior. }It involves a number of elements, including personal identification, language, thoughts, communications, actions, customs, beliefs, values, and institutions that are often specific to ethnic, racial, religious, geographic, or social groups. These elements influence beliefs and belief systems surrounding health, healing, wellness, illness, disease, and delivery of health services." - NIH As PAs, we should respect the culture, values, beliefs, and expectations of the patient

What is the definition of medical error?

the failure of a planned action to be completed as intended or the use of a wrong plan to achieve an aim An error is defined as the failure of a planned action to be completed as intended or the use of a wrong plan to achieve an aim. Errors can include problems in practice, products, procedures, and systems.

leadership style: the facilitator

the leader allows the GROUP TO RUN ITSELF and not intervene unless necessary

leadership style: managerial

the leader who sees herself as a manager is concerned primarily with the running of the organization

leadership style: history's slaves

the situation dictates that a person finds himself or herself LEADING OUT OF NECESSITY

What is PAEA vision?

to improve the quality of health care for all people by fostering excellence in PA education

What is the following example considered as? Patient received appropriate Tx with Abx for first time - developed allergic reaction unavoidable adverse event? Error of omission? Negligence? Ameliorable adverse event? Preventable Adverse events?

unavoidable adverse event

leadership style: passive resistance/setting the example

with self-discipline, self-control, and self-purification the leader is FOLLOWED BY EXAMPLE only, not as a person

If you change employment after you receive a limited permit, do you have to obtain a new permit?

yes

Can a limited permit be renewed?

yes, can be extended for one year if endorsed by current or prospective supervising physician (therefore the maximum time a limited permit can be held is two years)

Are cough medicines and antidiarrheals a scheduled drug?

yes; Schedule V

What are steps in disclosing error?

{past}Explain that an error occurred {past}Apologize and express regret for the error {past}Convey the underlying elements resulting in the error {now}Address medical ramifications (a consequence of an action or event) and how the error will be managed {future}Describe steps that will be taken to prevent the recurrence of the error {future}Arrange to speak again at a time that is convenient to patient and family {future}Talk about follow-up

What is the Supervisory ratios in NYS?

} Supervision (another big change in 2013) ◦ Supervisory ratios prior to March 29, 2013 outpatient setting 2 PAs: 1 supervising MD inpatient setting 6 PAs: 1 supervising MD correctional facility 4 PAs: 1 supervising MD ◦**Supervisory ratios after March 29, 2013 outpatient setting 4 PAs: 1 supervising MD inpatient setting 6 PAs: 1 supervising MD correctional facility 6 PAs: 1 supervising MD ◦Op and Cf increased by 2. *Hospital/nursing home

Understanding the Apology Which of the following is appropriate? A. ◦"I'm sorry if the nurse had only called me..." B. "I'm sorry for what has happened to you." C. ◦"Hospitals are complicated places...." D. ◦"I know this is unpleasant, awful, .... but believe me, for me it's shattering."

}"I'm sorry for what has happened to you." ◦Always appropriate }Do not blame "the system" or colleagues. ◦"Hospitals are complicated places...." ◦"I'm sorry if the nurse had only called me..." }Be careful of apologies that include "buts." ◦"I know this is unpleasant, awful, .... but believe me, for me it's shattering."

When is national PA week?

}**National PA Week - October 6-12

What is PANRE & Certification Maintenance?

}10-year cycles as of 2014 (used to be 6 years) - Five 2-year cycles for CME. - Each cycle starts May 1st (year of initial certification) and end Dec 31st - Log 100 CME hours Minimum 50 categories 1 . But can log 100 categories 1 instead Pay a Certification Maintenance fee PANRE is 4 hours - 240 questions

What is the prevention strategies for medication-related errors?

}5 Rights of Medication Administration ◦Right medication ◦Right patient ◦Right dosage ◦Right route ◦Right time (frequency) }Verify patient's medication - from patient, pharmacies, other clinicians, family members }Medication Reconciliation }Clear and detailed documentation }Use weight-based dosage - especially for kids }Be familiar with your institution's medication administration policies, regulations, and guidelines. }Have a drug guide available (PDR or electronic format - Epocrates) }Utilize Pharmacists }Use appropriate patient identifiers }Verify allergies and reactions

What are examples of necessary Confidentiality Breaches?

}A patient threatens serious self-harm or harm to someone else. }The patient is a suspected victim of child abuse or neglect. }The information relates to a crime. }The patient is not fit to drive but still request to drive

What should be done for disclosure of medical errors (from a team perspective)?

}A team needs to: ◦Understand institutional requirements for conducting the disclosure. ◦Agree on who will be involved in the disclosure. ◦Agree on what will be said. ◦Agree on how each team member's role will be represented. ◦Know what to expect from the patient and family.

How to be prepared for Compensation Questions?

}Acknowledge that the question is legitimate. }Explain that you are not qualified or authorized to address those issues. }Assure them that others who are qualified will speak with them in the near future.

What are the PA role in various practice setting?

}Across all medical, Surgical and other sub-specialties, PAs: Provide medical care and care coordination Perform physical examinations Diagnose and treat illnesses Prescribe medications Order and interpret lab tests Perform procedures Assist in surgery and enhance healthcare coordination

Fact about CHIP

}Administration varies from state to state }Usually provides coverage for basic services ◦Routine check-ups ◦Immunizations ◦Doctor visits ◦Prescriptions ◦Dental and vision care ◦Inpatient and outpatient hospital care ◦Laboratory and X-ray services ◦Emergency services

Employment

}After Graduation and complete all requirements }Cleared by PA School Loans, tuitions, etc., must be cleared }Take NCCPA certifying Exam Must pass the exam - for an unrestricted license }Apply for State Licensing Submit forms/Application }Most Importantly - APPLY FOR A JOB }Apply for National Provider Identification (NPI) number }Apply for Drug Enforcement Agency (DEA) Registration

What emotion may patient or clinician undergo?

}Anger }Sadness }Fear }Uncertainty

When does power of Attorney come in play?

}Appointed agent to act on behalf of a person who is incapacitated **Legal matters only! and can not be used for medical decisions ◦Buy-sell real estate ◦Conduct banking ◦Invest money ◦File lawsuits ◦Manage taxes

}Mr. Smith, a regular patient of Dr. Jones was seen at 8 am last Wednesday in the office by PA Persaud for hypertension. Dr. Jones arrived in the office at 10am and the case was discussed with him in detail. Which of the following is/are true? }A. This visit can be billed as an incident-to visit }B. This bill will be paid at 85% rate under the PA's NPI/name. }C. Dr. Jones will be reimbursed at 100% rate }D. The bill should be submitted under Jones' name and NPI number

}B. This bill will be paid at 85% rate under the PA's NPI/name. ** The physician is not on the premises }Criteria for Incident to: ◦The physician must be physically present on the premises at the time of care [Medicare ONLY!] - allow 100% coverage. -> Not necessarily be an active participant in patient's care. The exception to this ◦The first visit (new patients) must first be seen by the physician. [PAs to provide subsequent care for the same condition] ◦Established patients - new problems must be treated by the physician first. [PAs to provide subsequent care for the same condition/complaints]

HOw to cope with challenges?

}Be proactive }Be creative }Face conflict squarely }Always look for common ground }Retain your objectivity }Look for opportunities to collaborate }Listen }Ask for 360-degree feedback...and use it }Look at what's going on around you. }Reach out for help in facing internal challenges. }Create mechanisms to revisit your vision. }Share the burden. }Find an individual or group with whom you can discuss the realities of leadership. }Make sure you have personal time.

What is the PA-Nursing relationships?

}Both groups (NPs and PAs) Contribute significantly to healthcare Believes their roles are cost-effective solutions to the healthcare crisis. Work well as a team

What are reasons of PAs and PA-S doing CPT or ICD? [4]

}Bottom line -> to get the practice paid 1. ◦Allows tracking of outcomes 2. ◦Makes diagnoses and procedural patterns clearer across practice settings and insurances 3. For PA students - helps track what you've seen and done. Keep records of all you've done.

Memory trick

}CPT code - What you did (Procedures) - P for procedure }ICD code - What you found or diagnosed - D for diagnosis

What are the Physician Assistant Competencies? <6>

}Medical knowledge, }Interpersonal and communication skills, }Patient care, }Professionalism, }Practice-based learning and improvement, }Systems-based practice.

How many and what are categories of Current Procedural Terminology (CPT)?

}CPT codes are divided into three categories. }Category I codes: [For billing]. ◦Describe procedures that are accepted as standards of care codes and are also the primary codes used for reimbursement for physician and non-physician provider services from third-party payers and patients. }Category II codes [performance code] ◦Are used for performance measures and data collection. ◦ These are supplementary tracking codes - its use is optional }Category III codes [Research code] ◦Are temporary codes for emerging technologies, services, and procedures. ◦Ex: in Research - Purpose is to help researchers track emerging technologies and services.

Where are PAs in Rehab and Long Term Care?

}Care provided mainly to the elderly, disabled, post-surgical patients, etc. }Studies show that PA in LTC - decrease preventable hospitalization for residents }Manage acute and chronic conditions }Assess and manage new and current patients }Make rounds with a multidisciplinary team - including family members }Case management and Care coordination }Coordinate with attending to send patients to hospital or discharge to home }Duties similar to the in-patient hospital setting

What are different categories of CME?

}Category 1 ◦Pre-approved lectures, conferences, specific articles ◦AAPA, AMA, AAFP ◦Free CME sites - e.g. Medscape ◦Must log minimum 50 per cycle }Certificate Programs can be used for Category 1 ◦ACLS [ 2 days, can count 16 hrs, renew is 1 day - 8 credits], BLS, PALS, FCCS... etc. }Category 2 ◦Medically related activates that make you a better PA - Non-paid lectures, Journal Reading, Student Precepting (some schools do give Cat 1), Volunteer Work - Do not need to verify ANY Category 2 - Category 2 is not Mandatory - Can log 100 Category 1 instead - for a total of the 100 credits for the two year-period ◦Keep Good records of certificate Can be audited at anytime - Folder, CME - Anything log in NCCPA should put in a folder.`

Category 1 Codes

}Category 1 Codes - include six sections **◦Evaluation and Management (E/M) 99201-99499 ◦Anesthesiology 00100-01999 ◦Surgery 10021-69990 ◦Radiology 70010-79999 ◦Pathology and Laboratory 80047-89398 ◦Medicine Services and procedures 90281-99607 }Each carrier has specific reimbursement guidelines. ◦The use of a CPT code does not guarantee payment

ICD 10 characters for 1 - 6

}Codes have between 3 and 7 characters*** }Digits 4-6 provide greater detail of etiology, anatomical site, and severity }A code using only the first three digits is to be used only if it is not further subdivided. Example: Essential (primary) hypertension - I10 [No need memorize but know short as 3 and long as 7]

What is the PA-MD relationship?

}Collaborative and Collegial - (Clinical Colleagues) } Although, many laws use the term Supervision} Although the physician is ultimately responsible for the acts of the PA, the responsibility to ensure that PAs practice in accordance within ethical, legal, and medical standards is shared and reciprocal. } }It is the responsibility of the PA to seek advice and consultation when indicated.

Recap of Things needed for practice?

}Complete PA program }Take and pass NCCPA exam }Apply for State License }Apply for NPI }Apply for DEA certificate }Prescription privileges }Get Infection Control training }BLS and ACLS

Within how many days should you start NYS PEP and when should you retest blood borne pathogens?

}Complete course of Tx and f/u within 72 hours; retesting could be up to 6 months

What is contract and benefits?

}Contracts More common in private practice setting Also present in Facilities }A well written contract: Protects you and the practice from future problems You have to determine which works for you Things to consider are: an opportunity for growth, salary, your personal and family situations, etc. Is a contract right for you? }Facilities are investing money in you They don't want you to leave after training }Contracts would usually contain specific details regarding: ◦Salary ◦Vacation ◦Sick time ◦Medical benefits ◦Procedure when leaving - Amount of notice you should give - No-compete clause (cannot practice within certain miles of the practice) ◦Number of patients required to see, and other details as seen relevant by the practice ◦It is recommended to have an attorney look over your contract to make sure it protect your best interest.

Who holds the copyright of Current Procedural Terminology (CPT) and how many digits is all medical procedures?

}Copyright held by the AMA }Codes for all medical procedures ◦5 digits

What is coverage of Medicare Part B?

}Coverage includes ◦Medically necessary services ◦Preventative services ◦Medically necessary services Services or supplies that are needed to diagnose or treat medical conditions and that meet accepted standards of medical practice Example: medical/office visits, diabetes supplies, etc ◦Preventative services Health care to prevent illness (eg. Influenza vaccine) or detect it at an early stage, when treatment is most likely to work best. }Ambulance services }Durable medical equipment (DMEs) ◦Walkers, crutches, hospital beds }Mental Health services }Second opinions before surgery Covers Outpatient prescription drugs (Specialty drug - not cover your routine prescriptions: HTN, HLD drugs) Note: Original Medicare and most Medicare Supplement (or Medigap) ◦Don't cover hearing aids, routine hearing exams, or fittings for hearing aids

What is "Incident to" get 100% rate legally?

}Criteria for Incident to: ◦The physician must be physically present on the premises at the time of care [Medicare ONLY!] - allow 100% coverage. -> Not necessarily be an active participant in patient's care. The exception to 85% are ◦The first visit (new patients) must first be seen by the physician. [PAs to provide subsequent care for the same condition] ◦Established patients - new problems must be treated by the physician first. [PAs to provide subsequent care for the same condition/complaints]

What are other names for Cultural Competency?

}Cultural sensitivity }Cultural Humility }Cultural awareness

Where do you find jobs?

}Current rotations }York College email list }AAPA and NYSSPA websites }Internet searches/websites }Referral from friends, colleagues

What are two types of DNR and what is DNR?

}DNR Order instructs medical professionals not to perform cardiopulmonary resuscitation (CPR) }Hospital or Non-Hospital DNR

What are the key aspect of PA education in Maturation Period? Maturation Period Issues on Standardization

}Debate over the minimum academic degree to acceptance into PA Programs Small number of institutions offering Masters degree

Learning Objectives

}Demonstrate an understanding in providing medical care to patients from diverse populations } }Discuss the causes of differences in health status across differing cultures } }Analyze cultural factors impacting health and healthcare delivery } }Recall health/healthcare issues related to immigration and acculturation } }Provide specific examples of cultural competence in healthcare delivery } }Discuss the impact of socioeconomic factors on health } }Understand the role of effective communication between patients, their families and other health professionals }

Learning Objectives

}Describe the certification, registration, licensing, and recertification process }Recall the credentialing laws and regulations regarding professional practice }Recall the credentialing process and requirements }Understand employment contracts }Describe the various types of continuing medical education }Recall specific certifications available to the PA }Describe forms of medical malpractice insurance

Learning Objectives

}Describe the impact of HIPAA regulations on PA practice }Discuss patient confidentiality }Define informed consent }Describe and contrast advanced directives

What determines Medical Decision Making?

}Determined by: How many # of diagnosis? How much data reviewed? What is the risk level? }Number of Dx and Management options considered }Amount/Complexity of data reviewed Labs Prior records Consultant Reports }Risk Level Complications Morbidity Mortality

Who are STUDENTS in the 2000s - Moving forward?

}Diverse from all walks of life ◦Racial, ethnic, socioeconomic, geographic location }Different educational backgrounds Because PA programs were more competitive to get in

Who are students since 2010?

}Diverse from all walks of life ◦Racial, ethnic, socioeconomic, geographic location }Different educational backgrounds ◦Minimum Bachelors degree to enter (for those conferring Master's degree) }PA programs are now more competitive to get in

What are document required for health care proxy?

}Document Requirements ◦Name of person creating proxy ◦Name of agent ◦A statement that the agent will have the authority to make healthcare decisions ◦Signature with date ◦Two witnesses signatures with date Neither agent or alternative agent can be witnesses

What are elements included in each type of history?

}Each type of history includes some or all of the following elements: ◦Chief complaint (CC) - always required [Why here] ◦History of present illness (HPI) - must be done by the clinician - Components of OLDCART - Onset, Location, Duration, Characteristics, Aggravating Factors, Relieving Factors/Radiation, and Treatment ◦Review of systems (ROS) [Qs about each organ system][Pt come in abdominal pain, you would not ask neuro unless necessary] ◦Past Medical(P), family(F), surgical, and/or social history(SH) - (PFSH).

Medicare Will Largely Defer to State Law Requirements on How PAs Work With Physicians

}Effective 1/1/2020 - Medicare will defer to state law regarding supervision requirement (supervision vs collaboration) ◦Some states already removed the word "supervision" to collaboration }Hospice ◦In 2019, PAs were authorized to serve as "attending physicians," similar to physicians and advanced practice registered nurses (APRNs) under the Medicare program's hospice benefit. }PAs as Preceptors and PA Student Documentation ◦The work of PA students will be treated the same as that of medical students for preceptor documentation purposes. Preceptors can now review and verify, rather than re-document

Which of the following describes the term? ◦Providing care processes and achieving outcomes as supported by scientific evidence. A. Effectiveness B. Efficiency C. Equity D. Patient centeredness E. Safety F. Timeliness

}Effectiveness: ◦Providing care processes and achieving outcomes as supported by scientific evidence. *Treat with scientific evidence

Which of the following describes the term? ◦Maximizing the quality of care provided for a given amount of health care resources used. A. Effectiveness B. Efficiency C. Equity D. Patient centeredness E. Safety F. Timeliness

}Efficiency: ◦Maximizing the quality of care provided for a given amount of health care resources used. maximizing resource to provide quality of care

Which of the following describes the term? ◦Providing health care of equal quality to everyone regardless of their personal characteristics A. Effectiveness B. Efficiency C. Equity D. Patient centeredness E. Safety F. Timeliness

}Equity: ◦Providing health care of equal quality to everyone regardless of their personal characteristics Regardless of demography, everyone get same quality of care

}Established patient

}Established patient - 9921_ ◦Two of the three components must be at the same level or higher to support the level of care.

What is Equality in Healthcare?

}Everyone must receive the same level of care regardless of their race, gender, SES, immigration status, criminal history, etc. }As PAs/healthcare provider, you must see everyone as a "patient" that need your help. }It is important to engage in continuous self-evaluation for: ◦Implicit (unconscious) and explicit (conscious) bias

What are example of modifiers?

}Example: ◦The patient was seen in the office for URI (Office visit), but you also performed I&D on an abscess ◦Seen in hospital for abd. pain - had an endoscopy [warranted], cauterized a gastric ulcer, and also performed a colonoscopy [unwarranted] ◦Seen in office for routine physical (office visit), but you also removed ear wax.

What is the following statement describing? ◦Restrict enrollees to in-network providers Less selection ◦ ◦Lower cost to employers A. HMOs - Health Maintenance Organization B. PPOs - Preferred Provider Organization C. EPOs - Exclusive Provider Organization D. POS plans - Point of Service E. CDHPs (Consumer Directed Health plan or HDHP (High Deductible Health Plan)

}Exclusive Provider Organizations (EPOs) ◦Similar to PPOs

What should be told in medical disclosure?

}Facts - generally the sooner the better }Avoid natural desire to "put it all together"

Who were the educators in 90s?

}Faculty increased as a result of increased demands }Included more non-PAs with diverse backgrounds }Used a greater number of clinically based PAs as guest lecturers, mentors and preceptors

Which is this following statement describing? ◦Protects the government from being overcharged or sold substandard goods or services. ◦False Claims Act ◦Anti-Kickback Statute ◦Physician Self-Referral Law (Stark Law)

}False Claim Act (FCA) ◦The FCA imposes civil liability on any person who knowingly submits, or causes the submission of, a false or fraudulent claim to the Federal government. ◦Example: A physician submits claims to Medicare for a higher level of medical services than actually provided or that the medical record documents

Who governs fraud and abuse?

}Federal laws governing fraud and abuse ◦ ◦False Claims Act ◦Anti-Kickback Statute ◦Physician Self-Referral Law (Stark Law)

What if you make an error?

}Follow your institution policy }Be honest and disclose to your supervisor and supervising physician }Discuss as a team how to proceed }Also disclose to the patient American Academy of Physician Assistants—AAPA A physician assistant should disclose to his or her supervising physician information about errors made in the course of caring for a patient. The supervising physician and PA should disclose the error to the patient if such information is significant to the patient's interests and wellbeing. Errors do not always constitute improper, negligent, or unethical behavior, but failure to disclose them may."

What are four parts of medicare system?

}Four Part System }Medicare A ◦Inpatient (Hospital) }Medicare B ◦Outpatient (Medical) and specialty drugs }Medicare D ◦Drug program }Medicare C ◦"Opt out" ◦Advantage plans

What is PANRE focused on?

}Generalist Exam [everything] or }Practice-focused Exam [specialty focused] ◦Three areas of practice-focused. Adult Medicine Surgery Primary Care ◦60% from the generalist exam ◦40% from practice area ◦Allow PAs to focus on their general area of practice

PANCE registration

}Graduated from an ARC-PA accredited PA program }Registration to take PANCE }Register as early as 90 days prior to program completion [Different from graduation date] }Earliest can take the test is 7 days after program completion }Have 180 days from the beginning of your exam timeframe to take the exam }Can only take the PANCE Once in a 90 day period or 3 times in a calendar year. Can register 90 days before but can not take it till 7 days till program completion

What are all types of plans offered by insurance company?

}HMOs - Health Maintenance Organization }PPOs - Preferred Provider Organization }EPOs - Exclusive Provider Organization }POS plans - Point of Service }CDHPs (Consumer Directed Health plan or HDHP (High Deductible Health Plan)

What are strategies to prevent non-drug errors including teamwork-building?

}Hand-off communication }Involve multidisciplinary team }Collaborate to reduce infections }Avoid diagnostic errors }Utilize Electronic Health Record

What are strategies to prevent medical error (communication)?

}Hand-off communication ◦PA to PA sign-out ◦PA to RN, Physician communication ◦HOC section in EHR }Involve multidisciplinary team and utilize team-building skills e.g. pharmacist }Collaborate to reduce infections e.g. form committees & remind employees of the importance of these strategies (measures) - Advertise on hospital websites, paychecks, signs in hospitals, screen savers, etc }Avoid diagnostic errors ◦Incorporate the entire team (physicians, radiologists, etc) ◦Utilize expertise and skills effectively }Utilize Electronic Health Record ◦Not an issue anymore

What are 3 Components Determine the Level of Service? [Level 1, 2, 3]

}History - subjective info. }Physical - obj. info. }Medical Decision Making - A & P S/O/AP }Different requirements for new/established pts: ◦New patients - must document all 3 [SOAP] ◦Established patients - must document at least 2 of 3

Common vulnerabilities among the immigrant population How does Immigration Status affect immigrants?

}Immigration Status ◦Legal status prevents access to social services jobs with benefits healthcare coverage (lower rates)

What are the Benefits of IPC?

}Improved communication }Increased efficiency }Improve employee morale, increases inter-staff support, less turnover }Fosters creativity, ideas }Better problem-solving }Enhanced learning }Networking }Better clinical outcomes, cost-effectiveness, safety }Strengthened professional identity }Facilitated collaborative projects }Reduced competition & conflict, better conflict resolution

Where does PA serve as a team leaders?

}In many areas of practice (especially in patient-centered medical homes)

Who were the students in 90s?

}Increased demand for PAs }More applicants }Better qualified students

What is POS HMO?

}Individuals can decide to choose in network or out of network providers }In-network covered by HMO benefits }Out of network providers covered by fee for service. ◦Results in higher premiums but more choices for the consumer.

What are examples of QA in health care? [5]

}Infection control }Safety programs }Risk management }Credentialing }Peer review

How will ICD 10 help? [4]

}Informs better clinical decisions as better data is documented, collected, and evaluated }Provide greater specificity, laterality [L or R], and more detailed documentation of patient diseases }Allows better documentation of patient complexity and level of care, supporting reimbursement for care provided }May reduce audit risk exposure due to better documentation

What is POS plan licensed as?

}Licensed as PPOs

Common vulnerabilities among the immigrant population How does Limited English Proficiency affect immigrants?

}Limited English Proficiency ◦Healthcare provider does not speak their language ◦Communication is a key element in healthcare ◦Poor communication can result in .. lower satisfaction with care medication errors affect patient safety

Evaluation and Management - What is Time Based Coding?

}Many of the E/M codes have a time component associated with the code. }The time component is not applicable ◦Unless the encounter is >50% counseling and coordination of care. }In these instances, the time component of the code is used to determine the appropriate level of service.

What can be done to help vulnerable immigrant ?

}Provide Culturally Appropriate Services }Improve Language and Health Literacy Services }Increase diversify the Health Care Workforce -------------------------------------------------------------- }Provide Culturally Appropriate Services ◦Recognize cultural differences ◦Be respectful of and responsive to the health beliefs and practices of diverse patients }Improve Language and Health Literacy Services ◦Effective communication between healthcare professionals and patients ◦Make sure patients understand everything Utilize interpreter services Utilize teach-back method }Diversify the Health Care Workforce ◦Include healthcare workers from different racial and ethnic groups and cultures ◦Cultural competence training for all staff

Which is this statement describing? ◦Providers bill public plans directly (straight Medicare and Medicaid) ◦Or bill Managed Care Organizations (MCOs) - if contracted with the government }Out-of-Pocket }Private Insurance }Public Insurance

}Public Insurance

What are the learning objectives?

}Recall the Legislations regarding PAs }Recall specific legislation regarding PA practice in New York State }Describe the process of occupational regulation }Recall the range of prescriptive privileges

Learning Objectives

}Recall the requirements for ICD 10 Coding }Describe the documentation required for proper billing }Describe PA-specific Reimbursement }Describe various methods of reimbursement }Recall billing procedures, including documentation and coding

Learning Objectives

}Recall the various models of PA training }Recall the major categories of the Standards for an accredited PA training program }Describe the PA training program accreditation process

What are the common Cultural Factors that impact health & healthcare delivery?

}Role of family/community/religion }Views on health and wellness AND death and dying }VIew on Eastern/western/alternative/traditional medicine. }Beliefs about causes and treatments of illness, disease (physical and mental) }Gender roles and relationships }Sexuality, fertility, childbirth }Food beliefs and diet

Which of the following describes the term? ◦Prevent actual or potential bodily harm. A. Effectiveness B. Efficiency C. Equity D. Patient centeredness E. Safety F. Timeliness

}Safety: ◦Prevent actual or potential bodily harm.

What are Schedule III - V?

}Schedule III Lower abuse, efficacious Contains less than 90 mg of codeine per dose unit Vicodin, Codeine, Anabolic Steroids }Schedule IV Lower abuse, efficacious Valium, Xanax, Ativan, Versed, Klonopin }Schedule V Contain < 2mg/1ml of Opiates (codeine or opium) Ex: Hycodan contains 5mg per 5 ml of hydrocodone OTC or prescription Cough medicines, anti-diarrheals *lower the number more addicted!

How to get rid of "second victim syndrome"?

}Seek support, take time to heal ◦Risk managers and critical incident teams ◦Support groups ◦Your physician supervisor ◦Your PA colleagues ◦Other healthcare provider colleagues ◦Personal counseling }Become an advocate for others

What are Self-Assessment CME and Performance Improvement CME (PI-CME)?

}Self-Assessment CME ◦Participate in activities to assess your own strengths and weaknesses •Often multiple-choice exams •Eg. PA review book •Get Feedback •Available on the internet • }Performance Improvement CME (PI-CME) Compare current practice to national standards Implement change Evaluate impact of change

Common vulnerabilities among the immigrant population What is Socioeconomic Status (SES) of immigrants?

}Socioeconomic Status (SES) ◦Studies show that immigrants, in general Are less likely to graduate from high school and more likely to occupy service occupations

What is the four consumer perspectives of health care needs? [In term of health they may need] What does the patient want?

}Staying Healthy: ◦Need help with disease prevention and being healthy. }Getting Better: ◦Need help to recover from an illness or injury. }Living with Illness or Disability: ◦Need help with managing an ongoing, chronic condition or dealing with a disability that affects function. }Coping with the End of Life: ◦Need help to deal with a terminal illness.

Which three-way can be used to measure for 6 domains/properties of Quality?

}Structure, process, and outcomes - These are measures can be used to measure any of the 6 IOM quality domains (Effectiveness, Efficiency, Equity, Patient centeredness, Safety, Timeliness)

How is supervision achieved?

}Supervision may be achieved through such mechanisms as: Chart review and audit On-site consultation Periodic telephone contact Telemetry Shared practice days

What is 7th character is only used in ICD?

}The 7th character is only used in certain chapters to provide data about the characteristic of the encounter

What is EVALUATION AND MANAGEMENT CODES?

}The E/M section describes patient encounters with the physician/PA for all services other than procedures and miscellaneous services. }The E/M services can be either problem-oriented or preventive in nature.

What does PANCE look like?

}The five-hour PANCE exam includes ◦300 multiple-choice questions ◦Administered in 5 blocks of 60 questions ◦60 minutes to complete each block. ◦45 minutes allotted for breaks between blocks ◦15 minutes to complete the PANCE tutorial. }Computerized exam At testing centers (Pearson Vue)

Which of the following describes the term? ◦Obtaining needed care while minimizing delays. A. Effectiveness B. Efficiency C. Equity D. Patient centeredness E. Safety F. Timeliness

}Timeliness: ◦Obtaining needed care while minimizing delays.

What are two types of malpractice insurance and which is better?

}Two basic types ◦Occurrence - better policy - Covers incidents that happen during the policy period regardless of when the claims are reported. - Occurrence coverage provides protection for each policy period indefinitely [years down the line and get sued]. The better option ◦Claims-made - An incident must happen and be reported to the insurance company while the policy is in force. - Once the policy is terminated, coverage no longer exists. - If your policy is claims-made - you should consider buying your own additional occurrence policy

You are a PA on the floor and was called in to see patient with abdominal pain. The patient is on a contact precaution (although you are not sure what the contact was). You wore the proper gown and gloves and went in to evaluate your patients. After which you washed your hands with soap and water. Over the course of your shift, you saw and evaluated many other patients. However, over the next few days several older patients started to develop abdominal pain and watery diarrhea. You realized that you had previously taken care of these patients. }What do you think these patients have? }What could you have done differently?

}What do you think these patients have? ◦Likely same disease transmitted from the first patient via fomites }What could you have done differently? Also, clean the stethoscope using alcohol-based sanitizing wipes

Most research on the healthcare workforce has focused on physician supply (without factoring in PAs/NPs), what did their result predict for over expansion?

•Most research on the healthcare workforce has focused on physician supply (without factoring in PAs/NPs), resulting in: ▫Expansion of medical school programs will lead to More competition for clinical sites More competition for jobs

Why did APAP [PAEA] discontinued their services tp new programs?

•These services were later discontinued due to the rapid decline in new program growth •Early 1990s - APAP (now PAEA) created processes for new program support ▫New program workshops ▫New program consultation services

What caused the decrease of federal support for PA programs?

▫**Funding became specific Focusing on primary care and rural delivery ▫Clinical site expansion ▫Recruitment and retention activities ▫Curriculum development focusing on managed care and geriatrics ▫Less on program development/infrastructure ▫By the late 1990s - Federal support of primary care programs began to decline What terminated the funding or changed the way of funding? ~ intension of money was not fulfilled ~ students instead of going into PCP, goes elsewhere

How many programs are there as of 12/2020?

▫268 accredited PA Programs ARC-PA (Accreditation Review Commission on Education for the Physician Assistant)

What are the roles of PAs nowdays?

▫Clinical responsibilities** ▫Research and program development ▫Education

What is the comprehensive health manpower act of 1972 did to PA programs?

▫Comprehensive Health Manpower Act (Section of the public health act) authorized support for PA training Education of PAs for ambulatory medical care Recruitment of minorities and women into health professions Recruitment of larger numbers of residents from medically underserved areas Deployment of PAs to medically underserved communities

What is the newer models of PA medical training?

▫Increasing emphasis on interdisciplinary teams and greater integration of medical students, residents, and PA students on most patient care services Professionalism & Inter-professionalism

Who are }Support Staff AKA - clinical support staff?

◦"Assistants" PCAs (Patient Care Associate/Assistant) CNAs (Certified Nursing Assistant) ◦Provides support to Medicine, Nursing, Allied health professional and other ◦May not need licensing or registration by the state

What are the six key elements should be part of every State's PA practice act?

◦"Licensure" as the regulatory term ◦Full Prescriptive authority [ 2-5 + Marijuana s/p specialized program completion ] ◦Scope of practice determined at the practice level ◦Adaptable collaboration requirements [check again] ◦Chart co-signature requirements determined at the practice ◦No restriction on the number of PAs with whom a physician may supervise [Not in NYS, there is number limitation in NYS]

What is the definition of healthy people 2020?

◦"a particular type of health difference that is closely linked with social, economic, and/or environmental disadvantage. Health disparities adversely affect groups of people who have systematically experienced greater obstacles to health based on their racial or ethnic group; religion; socioeconomic status; gender; age; mental health; cognitive, sensory, or physical disability; sexual orientation or gender identity; geographic location; or other characteristics historically linked to discrimination or exclusion."

Which time period did the PA educational change happened that led to PA moving into different setting and speciality? ◦Mid 1960s through 1970s - Pioneer Period ◦1980s - Maturation Period ◦1990s - Boom or Bust time ◦2000s - Moving forward - futures 2010 and up

◦1980s - Maturation Period

Obj. 2 What are the five standards of ARC-PA accreditation?

◦A - Administration ◦ ◦B - Curriculum and Instruction ◦ ◦C - Evaluation ◦ ◦D - Provisional Accreditation [for initial accreditation] ◦ ◦E - Accreditation Maintenance [for reaccreditation]

Who is gatekeepers?

◦A primary care practitioner who authorizes all care from other providers - manage care and organize referral *More common in HMO.

What is managed care?

◦A system that tries to control health care costs while ensuring access and improving quality of care

What is a Stigma and Marginalization of the immigrant population? What contributes to the stigma?

◦A variety of factors can contribute to this: - Differences in appearance (for example, wearing a traditional dress) - Cultural and religious practices - Language barriers - Speaking with an accent (even among immigrants who speak English) - Skin tone }Example of Stigma - undocumented immigrants overburden the system and take away from deserving citizens What is true? - Studies show that undocumented immigrants utilize very little healthcare - Being part of a stigmatized group can make immigrants reluctant to seek care because of concerns about poor treatment

Which one is used for Diagnoses? ◦A. ICD 10? ◦B. CPT Code?

◦A. ICD 10?

What issue does PAEA started to address? What is PAEA function?

◦Accreditation and certification ◦CMEs ◦role delineation ◦The overall goal of improving quality healthcare

Which groups are developed in the period of maturation? ◦Health maintenance organizations (HMO) ◦Preferred provider organization (PPO) ◦Diagnosis-related groups (DRGs) ◦A and B ◦All of the above

◦All of above

Which this following statement describing? ◦Makes it a criminal offense to knowingly and willfully offer, pay, solicit, or receive any remuneration directly or indirectly to induce or reward referrals of items or services reimbursable by a Federal health care program ◦False Claims Act ◦Anti-Kickback Statute ◦Physician Self-Referral Law (Stark Law)

◦Anti-Kickback Statute ◦Example: A provider receives cash or below fair market value rent for medical offices in exchange for referrals

Which is this following statement describing? Which law is in place preventing it? Getting reduced or free rent in exchange for referral for a number of patients ◦False Claims Act ◦Anti-Kickback Statute ◦Physician Self-Referral Law (Stark Law)

◦Anti-Kickback Statute ◦Makes it a criminal offense to knowingly and willfully offer, pay, solicit, or receive any remuneration directly or indirectly to induce or reward referrals of items or services reimbursable by a Federal health care program

What is a process in the measurement of 6 IOM quality domains evaluation?

◦Appropriate clinician activities carried out to deliver care. }Examples: ◦Percentage of females of specified age receiving mammography. ◦Percentage of patients with asthma for whom appropriate medications are ordered. ◦Number of times adolescents are provided guidance on smoking avoidance.

How is quality assurance achieved by?

◦Assessing or evaluating quality ◦Identifying problems or issues with care delivery and designing quality improvement activities to overcome them ◦Follow-up monitoring to make sure the activities did what they were supposed to.

}Describe the PA-Allied Health relationships

◦Collaboration/Team approach

What is CAQs Prerequisites

◦Current PA-C certification ◦Possession of a valid, unrestricted license to practice as a PA in at least one jurisdiction in the United States or its territories, or unrestricted privileges to practice as a PA for a government agency. (Note: If a PA holds licenses in multiple states, all of the licenses must be unrestricted.) Unrestriction - meaning you can pratice in full extend.

What is NYS: PA Legislation & Regulations for Chart Co-Signature?

◦Determined by supervising physician and facility - Billing will be issue if not signed. Co-signature not required prior to the execution of an order but is required to signed eventually!

What is PDSA model?

◦Developing a plan to test the change (Plan) ◦Carrying out the test (Do) ◦Observing and learning from the consequences (Study) ◦Determining what modifications should be made to the test (Act)

What are benefits of PA and Physicians working together in surgical practice?

◦Efficient teams: See more patients Reduce operative and anesthesia times Decrease post-op complications Decrease patients length of stay in hospital

What are typical Duties include in outpatient setting?

◦Evaluation and management of new and recurring patients ◦Management of chronic conditions ◦Referrals to other specialties or hospital ◦Perform minor clinical procedures ◦Order and interpret medical tests, Prescribe medications ◦Involved in coding and billing ◦Practice management - supervisory roles, manage staff, inventories [May be involved in a leadership role! ] ◦Documentation - H&Ps, progress notes, consult notes, SOAP notes ◦Completion of forms - medical clearance, disability, etc [may be cumbersome] ◦Other duties directed by supervising physician

What is the decisions are based on ARC-PA'?

◦Evaluation of info on the application ◦Report of site visit evaluation teams Any additional requested reports or documents

What does the following statement entail? ◦Explanation of charges after evaluated by insurance company A. Capitation B. Fee for service C. Explanation of benefit D. Network

◦Explanation of charges after evaluated by insurance company

What are challenges with standardization?

◦Extending the length and cost would limit enrollees Can't afford to pay ◦ ◦Limit the cost-effectiveness of the program Less PAs to fill the healthcare gaps ◦ ◦Reduce the pool from inner-city or rural who otherwise qualify for admission Reduce distribution of PAs ◦There was no final resolution in this debate Estimated that master's degree would gradually increase

Which is this following statement describing? Which law is in place preventing it? Knowing to submit a claim that is not true/ ◦False Claims Act ◦Anti-Kickback Statute ◦Physician Self-Referral Law (Stark Law)

◦False Claims Act False claim act protects the government from being overcharged or sold substandard goods or services. ◦The FCA imposes civil liability on any person who knowingly submits, or causes the submission of, a false or fraudulent claim to the Federal government.

What are needed for State Credentials (Licensure)?

◦Graduation from an accredited PA Program ◦Pass the PANCE (administered by NCCPA)

Which company is the largest HMO in NYC?

◦HIP Health Plan of New York is the largest HMO in New York City.

What is Medicare Part A?

◦Hospital Insurance ◦Automatically enrolled at age 65 if criteria met **has limited time of stay. }Coverage includes ◦Hospital Care ◦Skilled nursing facility care ◦Nursing home care Provided more than custodial care is needed ◦Hospice ◦Home Health Services Physical therapy, occupational therapy

What are hospital privileges for PA?

◦Hospitals must: Develop procedures for evaluating the credentials of PAs who apply for privileges or employment Designate a staff physician to supervise any PA they employ Make known to the PA - appropriate bylaws/rules identifying practice limitations (if different than those written into state law, rules, or regulations)

What is not covered under blanket consent? ◦Placement of intravenous lines ◦Administration of medication ◦Intravenous hydration ◦Invasive contrast radiological studies ◦Phlebotomy

◦Invasive contrast radiological studies needs further informed and written consent. The following are covered by hospital admission consent: ◦Placement of intravenous lines ◦Administration of medication ◦Intravenous hydration ◦Non-invasive, non contrast radiological studies X-ray CT ◦Phlebotomy


Kaugnay na mga set ng pag-aaral

Pharmacology Chapter 15 with Book

View Set

ES U4 Chapter 13.2 Stream Erosion and Deposition

View Set

Bontrager Workbook: Chapter 3-Abdomen

View Set

Module 5 Introduction to Computer Hardware

View Set

Life Insurance Policy Provisions, Riders, and Options Chapter Quiz

View Set

7 characteristics of free enterprise

View Set

Business Logistics and Transportation Ch. 8 (McLaury)

View Set

Kata Penghubung (Konjungsi): Koordinatif (k.) dan Subordinatif (s.) - (1)

View Set

Southwest Airlines Public Announcements 2019

View Set